Naturopath challenge

Yesterday's piece about naturopathy lead to a nice conversation, but I realized we're missing something. We don't actually have a naturopath to defend the profession.

You see, if, for example, you are curious what your primary care physician might recommend to you, you can go to the American College of Physicians website, or the US Preventative Services Task Force to get an idea of what the standard of care and evidence is for screening, prevention, and treatment. What isn't clear to me is what a naturopathic primary care provider has to offer.

Here's my question, and I encourage any and all naturopaths to respond (anonymously is fine):

A 65 year old male comes to you for an initial primary care visit. What is your initial approach to the patient? Aside from what you may find on evaluation, what prevention and screening recommendations will you make?

One hint: you cannot answer, "well, it depends," because there are certain things that evidence shows should be done for nearly every 65 year old male.

Addendum

Let's make the clinical facts a bit more interesting and typical.

Let's say the patient has a history of diabetes, hypertension, and coronary heart disease, and had a drug-eluding stent placed in his LAD about one year ago.

Today's vitals are significant for a weight of 66 kg, blood pressure of 160/92.

His fasting blood sugar today in the office is 230. His LDL cholesterol level is 98.

More like this

I appreciate your blogpost, as it opens up the possibility for a real exchange that may bring to light some facts about naturopaths that are usually well-concealed (perhaps due to the relative rarity of NDs, or to their inaccessibility for those of us who must try and cover all medical visits with insurance).

I suspect, by the way, that primary care practitioners may sooner, rather than later, opt to be paid directly by patients, rather than by insurance companies, since they're struggling just to cover overhead and aren't able to sustain their practices on the poor remittances and cash flow caused by insurance company shenanigans. At any rate, they'll wind up facing some of the same benefits (and challenges) that I believe NDs find in not dealing with insurance claims.

I'm a little concerned by your last sentence, though. It seems arrogant, although I acknowledge that I may be misinterpreting the emotional coloring behind your words, since text doesn't always convey the complexities of human expression too well! There's something about the whole blogpost that has me suspecting you of an attempted "Gotcha!"

At any rate, I'm passing this along to one or two naturopathic doctors that I've met here in Vermont. Maybe they'll be intrigued, and respond... Hope so!

No NDs?

It's no wonder - you don't have many Homeopaths, Acupuncturists or Chiropractors either. Is it any wonder - you've published blog after blog attacking them on the most specious grounds.

And... wonder of wonders, you persist in representing your chosen system of medicine as "evidence" based - as though those using alternative medicine are operating on no evidence.

Now today's target is the NDs. Only nobody's reading your stuff any more except for your small coterie of attack artists - "Joe", and a bunch of other anonymous losers.

Keep blogging. Keep fooling yourself and keep trying to convince everyone that Acupuncture, Homeopathy, Osteopathy, Chiropractic are all woo nonsense.

They're not responding because they're busy with real patients who have already found out all the "evidence" they need about which system of medicine is helping them.....and which isn't

By ScepticsBane (not verified) on 12 Apr 2009 #permalink

There's something about the whole blogpost that has me suspecting you of an attempted "Gotcha!"

PalMD described the potential "gotcha" in the last sentence: namely, that "it depends" without any context or explanation of what values are conditional and on what is a very inadequate answer and that there are some things that are so universally necessary that not including them would be practically malpractice. (I could probably come up with a scenario in which almost any screening test you could name would be unnecessary or impractical, but the scenarios would be pretty forced. In the real world some tests are essentially always necessary.)

I should make this a little more interesting.

look for addendum above.

ScepticsBane wrote: "on the most specious grounds." Huh?

And Aviva Gabriel wrote: "some facts about naturopaths that are usually well-concealed." Well, let's light a candle in the dark.

Here's the essentially naturopathic per the 'most specious well-concealed'

[this is one of my favorite activities, exposing their 'occult vitalism' that is written into their oath per HPN,

e.g. http://queenannenaturalmedicine.com/naturopathicoath.aspx

and into their consortia standards

e.g., http://www.fnpla.org/philosophy.htm ]:

http://naturocrit.blogspot.com/2009/03/decoding-naturopathys-occult-sec…

And here's a great critical article about that article of faith,

http://www.vancouversun.com/Health/Naturopathy+main+article+faith+canno… [I'm so jealous that these words are not mine!!!].

-r.c.

His fasting blood sugar today in the office is 230. His LDL cholesterol level is 98.

". . . And in three days, this patient will be at death's door."

Sorry — my friends and I got into the Maker's Mark and had a House marathon last night.

I refuse to give the patient an elevated ethanol level

(if I can't have one, nobody can)

Ahh but it does depend...how are his conditions being managed. If he is previously diagnosed with DM II (I assume) is he on metformin? Insulin? other drugs? When was his last labs run - HA1C, CRP, etc. When did he have his last Colonoscopy and prostate exam?
Is he following up with his cardiologist?

I don't know anyone who had stents places that isn't on some sort of medication - including fish oil? is he taking anything for his HTN? at 65 years old, he is probably on multiple meds? Who prescribed them? Do I have permission from the pt to contact my colleague?

Perhaps your some of your skepticism as to what a naturopathic physician would do in primary care setting comes from confusing naturopathic physicians trained at fully-accredited 4 year medical schools (see Princeton Reviews Best 168 Medical schools book), licensed (in 16 states with 3 more pending) with people who refer to themselves as naturopaths but receive no such training. We are trained to follow the same primary care "standards of care".

From the standpoint of Naturopathic Physicians that is why licensing is so important - the public deserves to know if they are seeing someone who is trained in primary care or not. There cannot be ambiguity. If you are interested in more information here are 3 organizations you can check out - www.aanp.org
www.aanmc.org
www.cnme.org

Ahh but it does depend...how are his conditions being managed. If he is previously diagnosed with DM II (I assume) is he on metformin? Insulin? other drugs? When was his last labs run - HA1C, CRP, etc. When did he have his last Colonoscopy and prostate exam?
Is he following up with his cardiologist?

I don't know anyone who had stents places that isn't on some sort of medication - including fish oil? is he taking anything for his HTN? at 65 years old, he is probably on multiple meds? Who prescribed them? Do I have permission from the pt to contact my colleague?

Perhaps your some of your skepticism as to what a naturopathic physician would do in primary care setting comes from confusing naturopathic physicians trained at fully-accredited 4 year medical schools (see Princeton Reviews Best 168 Medical schools book), licensed (in 16 states with 3 more pending) with people who refer to themselves as naturopaths but receive no such training. We are trained to follow the same primary care "standards of care".

From the standpoint of Naturopathic Physicians that is why licensing is so important - the public deserves to know if they are seeing someone who is trained in primary care or not. There cannot be ambiguity. If you are interested in more information here are 3 organizations you can check out - www.aanp.org
www.aanmc.org
www.cnme.org

Perhaps you are correct about my confusion of real and fake naturopaths.

So, here he is, what are the standard evidence-based treatments that any PCP can do without referring? What screening is recommended?

I'll help you be more specific: assume that he is currently on no medications---he sorta fell off the face of the earth a couple of months ago.

". . . And in three days, this patient will be at death's door."

Sounds like the scenario I was imagining, in which one need not do any screening on a 65 year old man: Patient also has Buerger's disease. He has had bilateral upper and lower extremity amputations, a colon resection (now has colostomy) and resection of all but 50 cm of his small intestine due to vascular disease. His enabling and/or homicidal wife lights cigarettes for him and sticks them in his mouth. She refuses to stop. Don't bother with any screening tests. Don't even bother with getting vitals. Just arrange hospice right away.

I don't know anyone who had stents places that isn't on some sort of medication -

If fact, I see it all the time. The question is, what medications should he be on. It's not a hard question.

BTW, this is probably like a USMLE Step 2 or IM shelf exam level question. It can be easily handled by anyone who knows a little internal medicine. Please, don't blow it for the ND's. They deserve some time to put together their answer (although I usually have to answer it in 10-15 minutes with a patient in front of me).

I'm a little concerned by your last sentence, though. It seems arrogant, although I acknowledge that I may be misinterpreting the emotional coloring behind your words, since text doesn't always convey the complexities of human expression too well! There's something about the whole blogpost that has me suspecting you of an attempted "Gotcha!"

Welcome to med school and residency. Welcome to the practice of medicine, the land of the gotcha.

The pain of feeling stupid pales in comparison to the pain of harming a patient. So the impolite question-and-answer game will never end.

a little deference....It's Easter Sunday and I'm running out the door to church. I'll get back to your game later tonight after the celebrations.

Proud ND,

You can't say, "fully-accredited 4 year medical schools."

You can say, "fully-accredited 4 year schools of naturopathic medicine."

Medical schools are accredited by the Association of American Medical Colleges, which has not accredited any school of naturopathy.

What would you do? First you do the patient intake history and go over it with the patient. You don't assume anything. If the patient has previous medical records, you try to obtain them or send the patient on the chase.
You do a physical exam and labs. You repeat them if necessary. You have the patient bring in his meds for examination. You question the patient on lifestyle, diet, habits, supplements, sxs, what bothers them, and whatever else may pop into your mind from what is divulged. You don't believe everything the patient may tell you.
After the exams and labs, you make a preliminary assessement of what that information may tell you initially.
You schedule follow-up appt.'s with the patient and communicate as necessary with any other physicians that may be involved with the patient's care, so that there is less chance of conflict. You make an assessment of how willing the patient is in the compliance category and how much the patient actually wants to do for themselves. In other words, the attitude of the patient towards helping themselves get or be well. Belief systems and societal conditioning will play into this. How open is the patient to change?

One of the problems with medicine is that it tries to lump people in categories and then speculate on what happened in studies. Everyone is not the same and you have to gear toward that particular patient in front of you.

By Anonymous (not verified) on 12 Apr 2009 #permalink

I'm sorry, Anonymous, but without further information, if that is all you have to offer you can be sued successfully for malpractice.

The pain of feeling stupid pales in comparison to the pain of harming a patient. So the impolite question-and-answer game will never end.

If you think your instructors are hard on you, try the deceased's little kids. They don't say anything -- they just look at you in your dreams.

By D. C. Sessions (not verified) on 12 Apr 2009 #permalink

I don't know anyone who had stents places that isn't on some sort of medication - including fish oil?

Would you recommend fish oil for someone with a cardiac stent? On what basis? What is the data on fish oil in cardiac disease in general and stents in particular?

As a 3rd year MD student, I'll confirm that this question is basically a series family medicine rotation shelf questions strung together.

I almost reflexively answered in my post, but I decided not to spoil the fun for the NDs!

I'm dying to know the answer, myself! I've got a couple of ideas, but I will settle for bouncing in place as I sit on the sidelines watching the story unfold. :)

What would I do with the 65 yo male...who dropped off the earth for a few years?
I would take a complete history and vitals, HEENT exam, check for peripheral neuropathies, look at his feet.
Run a CBC, Comp met and lipid, HA1C as well as check PSA level and vit D levels.

I would counsel the patient on diet and lifestyle. And see if he was willing to do the hard work needed - not a lot of patients are, and that's ok. I'd make sure he was following up with a cardiologist (I do mostly women's health and don't pretend to be all things to all people and prefer collaboration).

I would consider Statins and metformin based on the numbers and dependent on our discussion around diet/lifestyle (as well as finances eating really well is expensive).
I would find out when his last colonoscopy was and last DRE (assume pt will need one if it's been over a yr) with heme occult...those will be needed along with a testicular exam.

This is far too much to do in 1 visit but I would order the labs and get the patient back in.

As for the previous comments about fish oil...you may want to look on pub med. My father's cardiologist in KC put him on fish oil 3g qd after his stents were placed. I later found out it's common practice for many cardiologists now (at least among those I've spoken with).

As for the previous comments about fish oil...you may want to look on pub med. My father's cardiologist in KC put him on fish oil 3g qd after his stents were placed. I later found out it's common practice for many cardiologists now (at least among those I've spoken with).

I have. You probably should have too before responding. It's quick and easy. The evidence is weak at best. To quote one review: "Taken together, trial data suggest that if n-3 fatty acids can be useful in certain populations to prevent restenosis, their effects are probably weak." Try putting "fish oil, PTCA" with a limit randomized controlled trial into pubmed. The result is 13 hits. A few of the early trials looked positive but nothing after 1995 showed more than a very modest and very marginally statistically significant benefit.

I would consider Statins and metformin based on the numbers and dependent on our discussion around diet/lifestyle

Are those the only drugs you would consider? Can NDs prescribe meds or would you refer him to an MD?

I'm curious about what PalMD thinks of one particular controversial screening test that Proud ND mentioned. And about whether PND knows which one I mean.

Make that 2 controversial tests, but i get your point. I was going to give the "correct" answer this morning, but a similar question is going up on sciencebasedmedicine.com later today, so tomorrow I'll tell y'all what a real doctor should do.

Was one of the controversial tests the vitamin D screen?

I'm less interested in the controversial things he ordered than I am in the things he missed completely - copout to cardiologist nonwithstanding, some aspects of coronary artery disease absolutely within a primary care doctor's scope of practice, regardless of how collaborative one wants to be.

:hops in place, in suspense!!!:

I guess I could try to play, myself. Apply that "Google U" degree where it best suits-- on an imaginary patient. ;)

And see if he was willing to do the hard work needed - not a lot of patients are, and that's ok.

Kinda goes without saying. We don't put guns to anyone's head.

Pet peeve: low signal-to-noise of alt med articles due to nice sounding but empty comments like the one above.

Morning rounds on a typical medicine service would grind to a halt if such drivel were tolerated.

Make that 2 controversial tests,...

Oops. Yes. I seem to be suffering from the usual specialist's error of concentrating on her specialty only. The test that I was focusing on was...but that would be telling.

I'm dying to find out more about a couple of points in the HPI and medication list, but will defer to clear the field for any more naturopaths that might want to play.

Hello, PalMD,

I appreciate your civilized inquiry into how a naturopathic physician would treat the patient you presented. I feel that as in anything in life, the more we learn from each other, the more we can appreciate and have tolerance and share and grow from each other. This includes MD and ND medicine.

As a naturopathic physician graduated from National College of Naturopathic Medicine in 1988, and having done a year's residency there in Family Practice I am happy to answer your relatively easy question. I see many diabetic patients who come with the complications oftentimes associated with that condition itself, or as part of the Metabolic Syndrome criteria which are aligned with insulin resistance and pre and actual diabetes.

My first office visit is 1.5 hours. In that time I do an extensive history of the patient and his conditions--learning when conditions began, how they have progressed, how he is being treated (type and dosage of meds), etc, the typical doctor stuff, including mental/emotion issues, life stresses and so forth. I look at any blood work or any medical records he brought in. I do a physical exam including vitals, heart/lung, neuro check on his feet, and any other pertinent PE based on office visit. If not PE in a long time, then the DRE and so forth is included.

As a naturopathic diabetic expert, I would do the following with him:
Blood work including CBC, CMP, LFT, Ferritin, GGT, F/T Testosterone, Thyroid panel, PSA, A1C, glucose, c-peptide, fibrinogen, random microalbumunuria, 25(0H) Vit D. I require all patients who come to me with diabetes to have a dilated eye exam no more than 6 months previous to our visit--this is because I am so successful in lowering glucose levels so quickly there is a risk of worsening of any established diabetic retinopathy. I have figured out a supplement regimen which prevents this worsening. I would send him for an EKG and cardiac work-up to discern what his exercise capabilities are; and a podiatrist if any foot ulcers, signs of Charcot etc are evident. Has he had a routine colonscopy yet?

I send him home with "Dr. Richard Bernstein's Diabetes Solution" written by an MD with Type I DM for 65 years, a diabetologist with whom I preceptored. He has to read at least the chapters on the diet. He does a diet diary for a week and records his fasting, and 1.5 hr post-prandial glucose levels as well. If he needs a new glucometer, test strips, lancets--I'll write scripts for that. Is he on BP meds; probably, right? I need to discuss with the patient if I am taking over as primary doc to deal with all his condition or if he wishes to continue to work with his MD--most patients, in fact, almost everyone, wishes me to take over everything. I would then increase his BP meds a little for right now. I'm not too concerned with his 230 blood sugar; with my protocol that will come down substantially and one more week is not a problem, although I tell him to guarantee me he'll keep himself fully hydrated. I'll send him away with blood work orders at local lab, diet diary, chart to record his blood sugars, prescriptions if necessary, Bernstein's book, a clearly detailed Treatment sheet.

He'll come back next week when I'll go over the blood work with him, describe the strict Low Card diet he must follow via a very detailed handout I have, which includes components of eating healthily as well. I also go over the supplements I'll put him on (vitamins, minerals, fish oils, herbs, accessory nutrients), recommend exercise (if he has been cleared by the cardiologist), stress relaxation techniques which resonate with him, if necessary. Let's say he's NOT on insulin, so I don't have to describe all the comprehensive ways I deal with that. I give him directions that if his blood sugars go down frequently below 100 mg/dl to call me and stop his Sulphonylureas (imagining him to be on Metformin and Glyburide). I recheck his vitals and his BP is lower, 140/82, which is okay for now. I will see him weekly or every two weeks until he is stable and we have dealt with all problems, which usually just takes 2-3 visits. We will lower or remove medicines, as necessary. Follow blood work every three months.

That's a rough and general guideline how a typical ND works with the above patient.

Mona Morstein, ND
Tempe, AZ

So far, I'm unimpressed, but we'll see...

I accept your challenge and I believe you are in my area. WNY and souther Ontario are my areas. NY is appallingly backward and has no license for Naturopathic Doctors because of the politics, MLM companies show up in droves in NY and of course there is no MD support to protect the public from scam artists selling garbage in a fancy packages and paying huge amounts of money for the hope to get well.

By Maria Crony ND (not verified) on 13 Apr 2009 #permalink

My first office visit is 1.5 hours.

I realize that this is a crass question, but how much do you charge? And what do you do if the patient has neither insurance nor money? What would you do for an established patient that lost his or her job and no longer has either money or insurance?

(The point of these admittedly rude questions is that naturopaths often go on about how greedy MDs are. Be that as it may, I doubt that a naturopath with a well designed web site advertising her services and a 1.5 hour initial visit is giving her services away or seeing many people who are not able to pay, whether directly or via insurance, quite a lot.)

"...a year's residency in Family Practice."

Seriously? Or "naturopathic family practice."

I'm troubled by the conflation of "medicine" and "naturopathic medicine." How are the public to distinguish the two? Does the distinction matter?

One of my patients was seeing a naturopath a few years ago, before I knew what that was. I asked, is he an MD? My patient insisted he was. Found out later he was an "ND."

Why do people go to schools of naturopathy rather than med schools if they're going to represent themselves as graduates of "medical schools" and "family practice residencies"?

"I have figured out a supplement regimen which prevents this worsening." How? By experimentation? Did you get informed consent for that? Did you publish your methods, to minimize the need for further experimentation by others in the field? Do you have such a duty?

"I give him directions that if his blood sugars go down frequently below 100 mg/dl to call me and stop his Sulphonylureas (imagining him to be on Metformin and Glyburide)."

Wow, metformin is a sulfonylurea? Glad the ND could teach me something...

My suggestion- time to re-take pharmacology 101.

By Internist (not verified) on 13 Apr 2009 #permalink

The arrogance of PalMD writing "not yet impressed" is the height of old time patronizing MDs thinking that they have the judicial right to judge other professions! I know this might come off crass, but please understand, MDs, my need to have you approve of my medical knowledge and capacities is absolutely nil; I frankly could not really care less if you approve of me or my medicine. I am simply exchanging data to folks who I thought cared to listen.

Let's look to the FACTS of DM. What percentage of TREATED DM patients in the US attain the three goals of A1C <7%, BP <130/80 and Cholesterol <200? Only 7%! So, we can clearly state that MDs do terrible work with DM. Leading cause of ESRD, blindness, amputations--that is due to UNCONTROLLED DM, not DM itself. MDs cannot control this condition and look at the devastating complications that causes to patients. I'm entirely unimpressed myself with MD care of diabetes, PalMD.

I charge a bit of money for my care and patients pay out of pocket to me, and then we send in their insurance to see if they can get any money back. Insurance almost always pays for blood work. Patients also have to spend their own money for recommended supplements.

Dr. Benway, yes, I meant Naturopathic Family Practice. I apologize for the mistaken term. I had one patient with diabetic retinopathy suffer "re-entry phenomenon" due to the supposed increase in IGF-1 as a response to rapid lowering of glucose and insulin levels. He needed laser surgery as a result. I have had three patients since with diabetic retinopathy and none had any acute worsening of their eyes, nor needed any care. They were put on a particular supplement regimen I devised based on the science of nutrient and herbal care for eye health. I certainly share this regimen with anyone who wishes to learn it, both in my profession and out of it. I have not published this, which I probably should, I agree. Technical writing is not my forte, I admit.

How can I justify my supplement regimen? Let's look at just a few studies..
tudies show Vit D and calcium lowers glucose and inflammation in IFG patients: Diabetes Care. 2007 Apr;30(4):980-6. Epub 2007 Feb 2.
Antioxidant nutrients can reduce diabetic complications: Treat Endocrinol. 2004;3(1):41-52.
Vit C/E, zinc, magnesium prevent glomerular nephropathy in DM2 patients: Diabetes Care. 2005 Oct;28(10):2458-64.
MV reduces BP in DM2: J Am Coll Nutr. 2004 Jun;23(3):272-9.
Mg, Zn, Vit C/E increase HDL-C and apoA1 so should be used in DM2 to aid lipid panels: Diabetes Res Clin Pract. 2004 Jul;65(1):21-8
Alpha Lipoic acid reduces blood sugars and prevents/treats neuropathy: Diabetes Care, 2008. Alsoâ Lipoic Acid in Health and Diseaseâ by Drs. Zimmer and Packer.
Chromium helps diabetics especially when insulin resistance is worse: Metabolism. 2007 Dec;56(12):1652-5.
Metformin lowers B12 in patients: British Journal of Diabetes and Vascular Disease, 2004.
Increased intake of calcium reduces B12 deficiency from Metformin: Diabetes Care,2000, Vol 23, Issue 9 1227-1231

There is science supporting my supplement choices. I did not include botanical medical articles and such as this reply is long enough. DiabetesCare, btw, is put out by the ADA which states clearly in their Feb 2008 guidelines that patients should not get any type of supplementation. Go figure that one out! Perhaps they don't read their own journal articles....

On the other hand, I could just have chosen to use Avandia, causing how many cases of CHF in patients....?

Yeah, I'm pretty secure in the value of ND treatment of DM. Whether PalMd is impressed or not!

Dr. Morstein

For some reason the previous email was chopped in the middle with the FACTS of DM--Here is the entire email.

The arrogance of PalMD writing "not yet impressed" is the height of old time patronizing MDs thinking that they have the judicial right to judge other professions! I know this might come off crass, but please understand, MDs, my need to have you approve of my medical knowledge and capacities is absolutely nil; I frankly could not really care less if you approve of me or my medicine. I am simply exchanging data to folks who I thought cared to listen.

Let's look to the FACTS of DM. What percentage of TREATED DM patients in the US attain the three goals of A1C <7%, BP <130/80 and Cholesterol <200? Only 7%! So, we can clearly state that MDs do terrible work with DM. Leading cause of ESRD, blindness, amputations--that is due to UNCONTROLLED DM, not DM itself. MDs cannot control this condition and look at the devastating complications that causes to patients. I'm entirely unimpressed myself with MD care of diabetes, PalMD.

I charge a bit of money for my care and patients pay out of pocket to me, and then we send in their insurance to see if they can get any money back. Insurance almost always pays for blood work. Patients also have to spend their own money for recommended supplements.

Dr. Benway, yes, I meant Naturopathic Family Practice. I apologize for the mistaken term. I had one patient with diabetic retinopathy suffer "re-entry phenomenon" due to the supposed increase in IGF-1 as a response to rapid lowering of glucose and insulin levels. He needed laser surgery as a result. I have had three patients since with diabetic retinopathy and none had any acute worsening of their eyes, nor needed any care. They were put on a particular supplement regimen I devised based on the science of nutrient and herbal care for eye health. I certainly share this regimen with anyone who wishes to learn it, both in my profession and out of it. I have not published this, which I probably should, I agree. Technical writing is not my forte, I admit.

How can I justify my supplement regimen? Let's look at just a few studies..
tudies show Vit D and calcium lowers glucose and inflammation in IFG patients: Diabetes Care. 2007 Apr;30(4):980-6. Epub 2007 Feb 2.
Antioxidant nutrients can reduce diabetic complications: Treat Endocrinol. 2004;3(1):41-52.
Vit C/E, zinc, magnesium prevent glomerular nephropathy in DM2 patients: Diabetes Care. 2005 Oct;28(10):2458-64.
MV reduces BP in DM2: J Am Coll Nutr. 2004 Jun;23(3):272-9.
Mg, Zn, Vit C/E increase HDL-C and apoA1 so should be used in DM2 to aid lipid panels: Diabetes Res Clin Pract. 2004 Jul;65(1):21-8
Alpha Lipoic acid reduces blood sugars and prevents/treats neuropathy: Diabetes Care, 2008. Alsoâ Lipoic Acid in Health and Diseaseâ by Drs. Zimmer and Packer.
Chromium helps diabetics especially when insulin resistance is worse: Metabolism. 2007 Dec;56(12):1652-5.
Metformin lowers B12 in patients: British Journal of Diabetes and Vascular Disease, 2004.
Increased intake of calcium reduces B12 deficiency from Metformin: Diabetes Care,2000, Vol 23, Issue 9 1227-1231

There is science supporting my supplement choices. I did not include botanical medical articles and such as this reply is long enough. DiabetesCare, btw, is put out by the ADA which states clearly in their Feb 2008 guidelines that patients should not get any type of supplementation. Go figure that one out! Perhaps they don't read their own journal articles....

On the other hand, I could just have chosen to use Avandia, causing how many cases of CHF in patients....?

Yeah, I'm pretty secure in the value of ND treatment of DM. Whether PalMd is impressed or not!

Dr. Morstein

Third Time's the Charm?

Let's look to the FACTS of DM. What percentage of TREATED DM patients in the US attain the three goals of A1C <7%, BP <130/80 and Cholesterol <200? Only 7%! So, we can clearly state that MDs do terrible work with DM. Leading cause of ESRD, blindness, amputations--that is due to UNCONTROLLED DM, not DM itself. MDs cannot control this condition and look at the devastating complications that causes to patients. I'm entirely unimpressed myself with MD care of diabetes, PalMD.

Uncle! Holy cow! I can't get one paragraph through. Sorry for the repetitive emails.

Let's try it a different way.

The point being that it's a FACT only 7% of DMs in the US treated by MD attain goals of ADA blood sugar values; BP in control and Cholesterol less than 200 mg/dl. Only 7%! Now, let me tell you NDs are NOT impressed with MD work like that (PalMD). As a result of uncontrolled DM they lose their kidneys, their eyes, their feet and legs, their lives. MDs do not do a good job of treating DM; they should welcome alternative viewpoints!

Dr. Morstein

I'm sorry, mona, but that last bit came across as word salad (and thought salad).

Your argument appears to be that because the evidence-based goals are difficult to attain, we should abandon them and give them twigs.

There is also reasonably good evidence that aiming for the goal has benefit in and of itself.

Mona Morstein,

I give you credit for stepping up to the plate knowing you would be critiqued. No one likes to appear ignorant. But if you can toughen up a bit, you won't mind it so much and you'll learn a lot.

"I don't know" is often the right answer.

Patients may suffer lasting disability or death when a doctor is wrong. So nevermind your ego needs while being pimped. Your priorities will appear skewed.

Whitecoat Tales asked: "Why don't we just treat NDs like international med grads? If you want to be a pcp take boards and do a residency."

Some of us would, if that were an option for us. We do not fear the challenge posed by the International Medical Graduate Examination.

By Anonymous ND (not verified) on 13 Apr 2009 #permalink

Some of us would, if that were an option for us. We do not fear the challenge posed by the International Medical Graduate Examination.

I'm glad you don't fear the challenge. But PalMD's challenge here was on the level of questions asked on the required board examinations. The ND's that answered, did not answer correctly even though this 'test' was open book.

...old time patronizing MDs thinking that they have the judicial right to judge other professions

It's not about fear, it's not about judging professions. It's about responsibility. Your responsibility to not hurt your patients through ignorance.

I've been wavering on whether to respond specifically to your challenge, or simply share my perspective with you. I'm not sure that either will be received openly, but I'd like to try the latter anyway.

The comparisons between MD's and ND's frustrate me. Not because one is better than the other, but simply because the two can't be compared in the way in which conventional medicine would like. And without measuring up to these standards, ND's will never be truly welcome and accepted.

As I see it, MD's would essentially require ND's to receive comparable training to gain acceptance as PCP's. If you are looking for clinicians to measure up to the standards set by your organizations and licensing boards, then yes, I can see that. But, what if there were other means by which a "physician" could be trained to assess, treat and manage a patient at a primary care level? I'm not talking about crystal healing and chakra reading either.

I see patients on a daily basis who have a myriad of primary complaints: hypertension, chronic migraines, chronic fatigue, endometriosis, peri-menopause, allergies, eczema, etc. I may draw from some of the same assessment tools as any MD would for assessment; hormone panels for the peri-meno patient, BP readings (and cardio workup) for the hypertensive patient, etc. But, I also look at other factors such as balance of omega-3 fats in a patient with inflammatory disease, or the presence of estrogen dominance in a patient with endometriosis. And, while the tools in my toolbox may not come with patient monographs (and patents), they have their place and prove themselves to be effective time and time again. And yes, I realize that an abundance of RCT's would legitimize our treatments, but we're working on that.

The point of my post is that I'd like to see MD's and ND's working constructively towards managed patient care instead of trying to prove worthiness. In Canada, ND's have just been granted prescribing rights by the BC government. This decision is controversial, but I believe this is the first step towards true integrated care in Canada. ND's aren't biting at the bit to prescribe drugs, but it will round out our primary care role by allowing us to do so.

Ask a dozen chefs how to make the best meatball, lasagna or chicken noodle soup, and you are guaranteed to get a dozen answers. The answer to whose is "best" will be in the hands of the patron. While each will be fed, not all of them will be nourished.

But, my biggest question is do you think I would do this if I weren't helping people? Do you think people would come to my practice, refer their friends and family if they weren't being helped? I was accepted into a conventional MD program, but decided to take the path less traveled. I haven't been disappointed. I see great value in all types of medicine, but we'll lose that value if we all try and become the same kind of doctor.

By Dr Jen, ND (not verified) on 13 Apr 2009 #permalink

Dr. J., I've question for you that is not about NDs vs. MDs, or patient care at all, but it is relevant.

It's a question I ask a lot of NDs:

does science contain the supernatural?

-r.c.

"Do you think I would do this if I weren't helping people? Do you think people would come to my practice, refer their friends and family if they weren't being helped?"

#46 I don't doubt for a second that you think you're helping people and that people think they're being helped, but the fact is that human perception is flawed and biased, and unless you can back medical claims up with statistically sound data, there is no reason to believe them.

As to your other question, what if there were other ways to take care of really common medical issues that might not need the same level of expertise an MD provides? Maybe we could come up with some kind of nurse, with expanded training and more difficult licensure, who might even be able to prescribe medications and admit patients... some kind of "nurse practitioner of medicine" or something.

And yes, I realize that an abundance of RCT's would legitimize our treatments, but we're working on that.

You should also realize the inadequacy of this argument. You don't form a conclusion, act on it, and then hope for data.

Science---you're doing it wrong.

But, what if there were other means by which a "physician" could be trained to assess, treat and manage a patient at a primary care level? I'm not talking about crystal healing and chakra reading either.

Why not?

Dr Jen, ND sez:

The point of my post is that I'd like to see MD's and ND's working constructively towards managed patient care instead of trying to prove worthiness.

Wow.

So it doesn't matter to you if the person treating you has proven themselves worthy, only that they are a team player? It doesn't matter if they know what they are talking about as long as they are friendly towards other professions? That sort of attitude is bad enough in an engineering organization, where a lack of quality engineers will result in crappy products (but where it is sadly becoming fashionable among the pointy-haired bosses). In medicine, that attitude can be lethal.

Yes, it is important for doctors to work constructively towards managed care. But I think it is *more* important for them to actually know what they're doing.

By Calli Arcale (not verified) on 14 Apr 2009 #permalink

I see patients on a daily basis who have a myriad of primary complaints: hypertension, chronic migraines, chronic fatigue, endometriosis, peri-menopause, allergies, eczema, etc...and, while the tools in my toolbox may not come with patient monographs (and patents), they have their place and prove themselves to be effective time and time again.

Dr. Jen, I find it very interesting that the majority of the 'primary complaints' you mention in the above quote are characterized by oscillating severity of symptoms. Can you appreciate that this fact alone makes robust RCTs critical for determining if any of your 'tools' are actually having a predictable, measurable effect?

By Jennifer B. Ph… (not verified) on 14 Apr 2009 #permalink

I can't help noticing, you've described a hypothetical patient who has been multi-diagnosed and inadequately treated (a scenario you say is very common for you to see in your practice), who then shows up at an ND's office for an alternative to the inadequate care he's received thus far (a scenario that we see very often also). Then you want to know if we know what you'd do if he came to see you?!

When we answer differently from your pat answer, you declare us 'wrong' and when we answer the same, you complain that we're not offering anything different. Hmm

From where I stand, I have to wonder why in the world would I want to do what has already been demonstrated in so many mainstream medical offices to be inadequate and unwanted... by simple fact of the patient standing in front of me, as opposed to going back to the original doc?!

I'm writing this fully realizing that I will not be heard. You're reading these ND posts trying to correct our reasoning and punch holes in our positions rather than to understand where we're coming from. That's fine if you just want to try to win a debate. However, there's not going to be a winner. We'll all go along believing as we did. And that's fine too.

Dr. Morstein offered you a little insight into a protocol that so regularly gets her pts to your target values that she's as surprised when the numbers don't fall as mainstream docs are when they do. Yet, you decided to 'correct' her instead of saying "I don't know, what are you doing that works?" As if what she sees in her office isn't real until she publishes... can't be real because you didn't learn about it in 'real' school.

May I suggest that for all of you on this blog looking for answers to the great mystery of "why do ppl go to them?!"; you need look no further. The simple answer to why we have patients is because they are looking for medicine that works. The reason our profession exists at all is because ppl want medicine that works. Ppl want to be treated in their entirety instead of being looked at through a specialty lens or being treated like the things important to them are drivel.

You look for evidence in journals because you seem to believe that somehow our flawed human perception is corrected when it's written in a peer-reviewed format, meanwhile, the evidence in the office is saying that the system is broken. The journals are nice AND, you have to admit, if you actually waited for them to provide all the evidence you say you need, there'd be very few treatments or protocols in your toolbox. Good science also attempts to describe what is observed in real life... this is one of those areas where the medical arts clash with the medical sciences.

If you really want to end the use of alternative medicine in this country, my suggestion is to get your own house in order rather than using all this energy preaching to the choir or trying to 'fix' ND's and your pts. First of all, we're not broken. 2nd, people leave mainstream medicine looking for help only when they aren't getting it in the first place.

Let's face it, we are coming from different paradigms. I can agree that you seem to get results with some of your pts and seem to drive others away. I can also agree that ND's do the same. You'll have judgments about the one's that leave you and come to us and we'll have judgments about the one's that go back to you. You'll claim repeatedly that we're dangerous, despite your lack of evidence. And we'll continue to practice good naturopathic medicine based on our model anyway!

By William Franklin (not verified) on 14 Apr 2009 #permalink

I am merely a second year medical student but there are a few easy things here that scream out to me. I am sure I will miss something as I always seem to but here it goes.

I would try to get him on a diabetes plan including:
1) statins & ezetimibe to lower LDL cholesterol with an optimal goal of less than 70. Monitor his LFTs.
2)Blood Glucose management plan starting out with a bigaunide (metformin) unless ruled out by kidney function. May have to ramp up therapy to include insulin (Depending on what is covered by his insurance glargine as a long acting and aspart as a short acting though NPH could work as well for the long acting). Goal of a1c under 7%
3)80 mg of ASA a day.
4)if he smokes, smoking cessation.
5)try to get blood pressure below 130/90 start out with a ACE inhibitor/Angiotensin Receptor blocker as first line as they have benefits for diabetics
(those are the 5 stars in the five star plan that I've seen when doing my days in clinic)

Since you mentioned weight as significant I would guess that his BMI is over 25 so I would suggest weight loss. That would also help his hypertension and diabetes. Having him eat healthier would likely be a good suggestion.

Tests/Labs/Consults:
Stress test, with imaging after (either echo or dye)
Possibly an echocardiogram too (not sure if this would be duplication or not)
EKG
Colonscopy (if not done with in past 10 years)
Fecal Occult Blood testing
PSA
Prostate Exam
CBC
Metabolic panel
Hemoglobin a1c
lipid panel
oxygen saturation
urinalysis (looking for diabetic nephropathy)
consult with opto
single filament test (for sensation of his feet)
Pneumovax and influenza vaccine.
H. pylori test (if he has any symptoms of ulcers)

that is a quick start. I am sure I missed something vital but I think I covered the diabetes and heart management decently, and I remembered a few key things relevant for his age group.

By student guy (not verified) on 14 Apr 2009 #permalink

Mr. Franklin, your reasoning is rather anemic (but I'm sure there's an herb for that).

The fact that a patient has multiple medical problems does not mean that science-based medicine has "failed" them any more than it means they have small feet. It is what it is. DM, HTN, CAD are all very common. There is strong evidence available about how to properly treat these conditions.

So far the ND's have come up with, "but you didn't fix everyone perfectly", and "we have lots of things that work great but just cant prove it yet".

These are, for patients who are counting on us, very unsatisfying responses.

It is difficult, I know, when the truth is not congruent with your philosophy---just ask Rush Limbaugh. Still, to quote Jon Stewart, "it's supposed to taste like a shit taco."

My advice...either learn to accept reality, or learn to put extra salsa on the taco.

William Franklin,
I don't agree with the basic point of your post - people go to non-MDs because western medicine has failed them. I think you are closer to the truth when you talk about people choosing "different paradigms", and not wanting "being treated like the things important to them are drivel." In other words, I think the altmed community has created a space where people go to feel validated in what they believe. Western medicine doesn't do that. It says, 'Your blood pressure is too high, and if changing diet, etc. hasn't lowered it to reasonable levels you should consider strongly taken BP lowering medication. If you don't lower your BP you will be at risk for certain diseases.'
There's no validation in this approach, just a simple outline of well documented risk factors. BTW, you ought to take a peek at Strunk and White on the use of apostrophes.

Who is this internist? The REAL question is why does the patient have uncontrolled diabetes and CAD. Is it because the conventional medical system of preventative medicine is a complete failure? When most (not all) medical doctors M.D.'s can't treat themselves successfully without symptomatic tx, then how will they prevent their patients from spiraling downhill. The patient should have never reached the point he is at, period. It's a typical case of the blind leading the blind.

Let's look at the physician who posted this blog. He looks overweight, probably on a few anti-hypertensives to control his high blood pressure. He is likely on a statin since he believes drugs are the cure-all for disease despite the debilitating side effects. The fact of the matter Doc is you guys are on the way out. You have failed for over a century to provide health to the public. Good luck in the future, I always need another person to wash my car.

Patient of Naturopathic medicine

By Craig Wanton (not verified) on 14 Apr 2009 #permalink

The plain and simple truth is that patients do come to us. We exist because patients demand that we do. Patients come to us because the current medical system is broken. They are looking for answers. They are looking for help in attaining health. We give them that. You all can debate this topic for the next 20 years, however, the simple truth is that the face of healthcare is changing. Patients are demanding it.

@Misha:
Patients like weed and vicodin too, but we don't necessarily give it to them.

@Craig:
I don't blame my patients for being unhealthy. I encourage them to have healthy behaviors, but being a good human being and a good doctor, I do not assume that because they have a disease that they have a moral failing. People who judge patients for being unable to fully control their own risk factors are, what we call in the business, total fucking assholes.

How else would you explain a pt with all these serious diagnoses which he got from somewhere and no tx plan, not even the drugs he "should be on"? There he stands in your office having had a stent and common diagnoses which have strong evidence for their proper treatment and yet he hasn't been treated.

Of course he's hypothetical but you've said you see this all the time clinically. If the system is working, how do these ppl keep showing up in offices needing the basics of care?

And, I agree it isn't always that mainstream medicine failed, meaning the docs didn't do a good job. I believe docs of all kinds are doing the best they can. It's probably more accurate to say that the insurance dictation of how medicine is practiced drives more ppl away.

Docs are required to see too many pts in a day to make the ends meet due to reimbursement problems and protocols are too dependent on the coverage they have rather than what is really in the pts' best interests. Docs frequently have to choose to fudge billing codes to get the tx they want to provide or give what they feel to be inferior tx.

Pts feel lost in the rush and things often go unsaid in the hustle of the visit-- things which the doc knows but doesn't have time to explain. I've been on both sides of that fence and it is unfortunate for all involved.

And I'm also not saying that we get it right all the time either. With that said, it's wishful thinking to believe that pts coming to an ND don't get real results and reduce risk factors.

I do feel that between mainstream med and naturopathic med, more ppl get better than with either one alone. I'm not trying to get rid of mainstream medicine. I'm merely pointing out that while you're busy trying to stamp us out, you're ignoring some very important issues within your own community.

As for my use of paradigm, I use it specifically as Kuhn presented it. And the comments about reality make my point perfectly.

'Dacks', I enjoy my punctuation! Don't you?

I am an ID doc, and I can't answer the question as to how to manage the patient as a PCP.

But I can reply to the issue of MD/ND co-existing and ND's being a PCP.

Patient of mine with chronic sinusitis saw an ND.
The therapies given for the sinusitis were IV DMSO chelation and it was suggested she wear wet socks to bed to draw the inflmmation out of her sinuses, which from a web search appears to be a standard ND treatment.

If you expect me to allow them what propose this kind of therapy to be allowed to be PCP's, you ask too much.

"the one's that leave you and come to us and we'll have judgments about the one's that go back to you."
I guess using standard punctuation is too rigid. Must be my cultural indoctrination coming through.

OHHH!!! I get it, Mark!!! This is where "integrative" health care comes in! The ND gets paid for telling the pt to sleep in wet socks, and an MD gets paid to cure the athlete's foot! ;D

I am ashamed at the remarks made by the MDs on this blog. As a second year medical student I cannot believe what I am reading. My school has a club for students interested in CAM and it is extremely popular. From my perspective, mainstream medicine is moving towards a balance between meds/surgery and botanicals/supplements.
My boyfriend is also an MD and he takes flax, wheatgrass and st. john's! NDs, please know that this does not represent our profession and I hope one day to work in an integrative practice.

By Caitlin Feinberg (not verified) on 14 Apr 2009 #permalink

From my perspective, mainstream medicine is moving towards a balance between meds/surgery and botanicals/supplements.

That may be the case. But it should only be moving there where we can produce evidence of value. For example you mention flax and st john's wort - both of which legitimately have some science behind them.

On the other hand, before jumping into practice with NDs I suggest you read about what their standard of care is. As a future MD you have an obligation to your patients to be aware of what treatments work, and what treatments are potentially dangerous.

I'm a med 3, and I can't believe that you're so glibly accepting that NDs are appropriately trained before investgating that claim. My institution also has an interest group for CAM. It's very popular. But thats not evidence that CAM works!

First, my party affiliation: I am a licensed naturopathic physician in the state of Arizona. Second, I'd just like to say that I'm enjoying reading this debate very much and I'm glad that the internet now makes such broad communication possible. Third, I'd like to say that all my colleagues really need to tell any skeptics out there is to stop by our offices and look at the charts. The numbers are there. The lab values normalize in our patients because our treatments work. Do we help 100% of our patients? Well, no, of course not. What physician does? Do we get 100% of our patients to be compliant with the treatments that they need? No, of course not. If we could accomplish that, we would have solved one of the great mysteries of medicine for all physicians everywhere. But, we have plenty of patients who do comply and plenty who do get better, get off their medications, and return to living normal, healthy lives that they never dreamed possible. Yes, there are less and more effective NDs just like there are less and more effective MDs. And, yes, our profession suffers due to the lack of funding for large-scale studies and long-term residencies. However, I am always surprised at the continuing debate about our medicine when the evidence already exists. What we really need is the funding for the research and publication of the existing evidence found in the thousands and thousands of patient charts in our offices and clinics across the country. I think that would lead immediately to more funding for more and better research, and for the extended residency education that most of us would welcome. Our medicine is very complicated, the protocols are far from standard, and the investigative research required to determine the causative factors in complicated cases can be enormous. Thank you, PalMD, for being willing to examine our profession and learn about it. Please encourage your colleagues to do the same. I know that we will all benefit in the future from more and more interaction between all of us in our quest to find what works best for each patient in the variety of situations in which we find them.

By Tiffany Taylor (not verified) on 14 Apr 2009 #permalink

Tiffany Taylor,

You state the research basis for naturopathy is poor.

You state your therapies work.

In science, that would be, "overstating the data."

I think you're the one being glib, Whitecoat. I went to undergrad at ASU (AZ has a copious amount of NDs and an osteopathic school). For American Med Hist I wrote a paper on the difference between DOs, MDs and NDs because I too wasn't sure what legitimate training NDs had. My father is a DO so I already knew the differences in their education.
Naturopathic Doctors are not the same as people that call themselves naturopaths/healers/health consultants and get some mail order or online degree. I too think that those individuals are dangerous. NDs go to four year naturopathic medical schools which are approved by the federal government, take licensing boards and often do residencies (some in allopathic settings...I think Cancer Treatment Centers was one place--is this still true?).
If you truly care about patient safety you would put your blogging efforts toward diploma mill degrees and unregulated medicine. If I practice in an integrated setting it will be with a trained and licensed ND, acupuncturist or herbalist.
Btw...I came across this conversation because I was googling for an ND near the twin cities for my CAM-minded aunt.

By Caitlin Feinberg (not verified) on 14 Apr 2009 #permalink

Caitlin, your naivetee is disturbing, but thankfully you are still young and early in your training. If you keep your mind open to evidence and rather more closed to ideology, you will learn much.

Bucking those in your profession who have been at this longer and know the evidence does not make you a maverick...it makes you dangerous, but at least there's time.

For example you mention flax and st john's wort - both of which legitimately have some science behind them.

Really? Last I heard, every well-designed study of SJW had more or less been a flop

Naturopathic Doctors are not the same as people that call themselves naturopaths/healers/health consultants and get some mail order or online degree.

No, they're the people that advertise things like homeopathy, bioidentical hormones, and vertebral manipulation on their websites (on which they also conveniently provide "a full dispensary to ensure where patients are able to obtain the highest quality nutrients in a convenient place, bypassing all the confusing and at times untruthful hype and hyperbole which patients face seeking help in various Health Food Stores and on the internet.").

If it walks like a duck and sounds like a duck...

By Jennifer B. Ph… (not verified) on 14 Apr 2009 #permalink

The fact that the scientologists are running the naturopathy schools in Britain is more than enough proof for me that they're nonsense.

By Marc Abian (not verified) on 15 Apr 2009 #permalink

Let's approach this from a biochemistry point of view. So this guy has diabetes type 2 and HTN. He has developed CAD due to plaque formation and now has a stent. What is the best way to tx? You guys go for the statin, Ace inhibitor, HCTZ, metformin "at least" which is fine for a short while. You'll likely tell the patient to exercise and eat healthier (DASH diet) but nothing really specific (you may even start with this first x 3 months). You go on your way as if you changed a life. Again what is the best way to tx? This guy is diabetic which has led to his CAD. His diabetes is the MAJOR contributing factor to his elevated lipids. How? HMG COA reductase is the enzyme in our liver that produces the major blunt of our cholesterol. It is turned on by insulin which spike due to high carbohydrate meals. This guy has high insulin levels due to his diabetes along with hyperglycemia. His pancreas continues to push out insulin which does not address the hyperglycemia in his blood. This excess insulin in the blood turns on his HMG COA reductase leading to elevated lipids. If you get the insulin levels, blood sugar under control the lipids will fall, period. Some feedback inhibition is lost due to the continuously elevated insulin levels. (excess cholesterol will feedback inhibit HMG COA reductase in healhty person). The best way to accomplish this is to put the patient on a very, very low carbohydrate diet. This will prevent glucose spikes in the blood which will prevent insulin spikes which will prevent HMG COA reductase from being turned on which will decrease cholesterol production. Once hyperglycemia and high insulin levels drop his body will begin gluconeogenesis, glycogenolysis, and lipolysis for energy. Glucagon will inhibit HMG COA reductase. Put him on high psyllium fiber which will remove bile and prevent enterohepatic recirculation. Prescribing statin should be reserved for patients who refuse to make the diet change to avoid carbs. Also, statins inhibit isoprenoid formation which inhibits COQ10- ubiquinol formation as well as hormones. The fatigue, cramps, and weakness is b/c without COQ10 we cannot donate electrons via the electron transport chain. Our cells become deficient in ATP and we get fatigue and malaise. I know you have heard of rhabdomyolysis. Hypothesis: Possibly combining cellular toxicity (due to low COQ10 which results in low ATP) and we all know that people process drugs at different rates via the liver cytochromes. Basically, if someone gets too high a dose of statin and they have lower than average cytochrome activity (phase 1,2), equals mitochondrial toxicity = cell apoptosis = rhabdomyolysis. His blood pressure is elevated but as long as he doesn't have signs of end organ damage (eyes, kidneys) I would try other methods before jumping into the diuretics. I know you are not interested in actually learning how inept you are (prevention-wise) so I will not go into tx since you obviously have no care in actually helping your patients to stop the progression of disease. But, I have a challenge for you Doc if you are man enough to take it. Go shadow via an accredited ND clinic and then talk your bull, if you still feel the way you do. This is a challenge to all MD's who read this by the way. Ignorance does not become you.

John Williamson

By John Williamson (not verified) on 15 Apr 2009 #permalink

Thats a whole lotta biochemistry, got any clinical trials?

Extraordinary claims demand extraordinary proof.

On the other hand, there is lots of evidence that thiazide diuretics are cheap, effective ways to reduce blood pressure.

... Ignorance does not become you.

Full of sound and fury, signfying nothing.

Incidentally, I have attempted to contact a local naturopathy practice. They were reluctant in the extreme to even talk to me. Which is suprising since all of you N's here have shouted that if only we saw you in practice we'd believe!

If they'll let me shadow, I will. I'll let you know what I find out

NDs go to four year naturopathic medical schools which are approved by the federal government, take licensing boards and often do residencies...

You say that as if "approved" has some clinical significance. In this context, not so much.

If the naturopaths would stop lying I might warm to them.

But to say that vitalism is supported by science is a lie. And selling the supplements you prescribe out of your own office is sleazy.

This response is to Whitecoat Tales.......Practicing medicine via Biochemistry is the only clear way to bring health to patients. It is the foundation of human health.

Clinical trials?

How long was it known that folic acid prevented abnormal folding of the neural tube? Well, they had evidence since the 50's. How many children have had to suffer because of morons in the science community who failed to recognize the importance of folic acid? The importance of folic acid by the FDA wasn't actually instituted till the 90's. Thousands of lives destroyed. I would be pretty upset if my child had spina bifida which could have been prevented, wouldn't you? Besides there are lots of clinical trials, RCT's, on CAM on countless herbs, and supplements. Look for yourself.

"signifying nothing"?

I have signified everything actually that food, nutrients activate enzymes in the body which can be manipulated via simpler, safer, more long term tx's rather than drugs. Drugs are great accomplishments of science but should rarely be firstline therapy. It is well known that nutrients are coenzymes that are absolutely essential for enzymes to function.

On the otherhand, I am happy that you have made the effort to shadow an ND. But before you proceed make sure they have graduated from an accredited school and optimally it should be in a licensed state. You would be gladly accepted to shadow at one of the accredited school's clinics. We have had several MD students rotate via. Also, you could attend the AZNMA, tx's centering around naturopathic medicine.

As for your colleague, PalMD, he is lost and will succomb to the common diseases of lifestyle and will ultimately be on > 10 drugs before his demise which will be long before the average lifespan.

John Williamson

By John Williamson (not verified) on 15 Apr 2009 #permalink

Practicing medicine via Biochemistry...

There is significant evidence against the interventions that have been suggested by the naturopaths here. That evidence trumps whatever biochemical theory you'd like to throw out here.

If you are suggesting that the interventions suggested by your colleagues is not the same as those that have been disproved, than why not publish your results?

Besides there are lots of clinical trials, RCT's, on CAM on countless herbs, and supplements. Look for yourself.

There are. Most of them are negative. On CAM. On countless herbs. On supplements. I specifically see no positive evidence of diet controlling diabetes with extremely low carb diets.

The Ns in question graduated from whatever school in AZ you guys seem to call home and purports to be licensed to dispense meds in arizona. The fact that they are practicing in the midwest doesn't appear to affect this advertisement. So far they are not impressing.

What, no comment on folic acid? Anyone?

Do you MD's ever consider the origin, cause of disease before people have markers, or values, or symptoms? How do people develop diabetes in an MD world? Have you guys ever ever thought of why and how it could have been prevented? I know you have never questioned how your medicine works but since you are too busy to look for alternative medicine which is loaded with positive information and signifcant values,

Do Low-Carb Diets Help Diabetes?
Small Study Shows Restricting Carbohydrates Reduces Need for Medications
By Salynn Boyles
WebMD Health NewsReviewed by Louise Chang, MDMarch 15, 2006 -- Should people with type 2 diabetesdiabetes follow very low carbohydrate diets? The American Diabetes Association (ADA) says "no", but a small study from Sweden suggests such a diet may be one of the best ways to manage the disease and reduce the need for medication.

In the study, 16 obese patients with type 2 diabetes followed a calorie- and carbohydrate-restricted diet for 22 months. Most showed continuing improvements in blood sugar that were independent of weight lossweight loss; the average daily dosage of insulin among the 11 insulin-dependent patients was cut in half.

"Many people are essentially cured of their [type 2] diabetes by low-carbohydrate diets, but that message is not getting out," says low-carb proponent and biochemistry professor Richard Feinman, PhD, of the SUNY Downstate Medical Center in Brooklyn, N.Y.

While agreeing that carbohydrate restriction helps people with type 2 diabetes control their blood sugar, ADA spokesman Nathaniel G. Clark, MD, tells WebMD that the ADA does not recommend very low-carb diets because patients find them too restrictive.

Okay, back to me, your own colleague here Clark MD completely writes off all diabetics, says its too hard to eat right to be healthy. That's it write off everyone, prepare for blindness, kidney failure, neuropathy, CAD, and amputations b/c its just too hard! Oh yea, decreased carbs equals decreased cholesterol, too! Go shadow someone, learn what you can to help people to the fullest. Use drugs when you need them, integrative medicine first.

John Williamson

By John Williamson (not verified) on 15 Apr 2009 #permalink

John Williamson: Hear, Hear! I echo that challenge! Just be aware that these parts only accept clinical based trials that fit their own agenda. You also have to remember their scientific creeds were spawned from greed and simple biochemistry is apparently not a science with them anymore.

Whitecoat Tales: I am impressed that you are attempting to shadow a ND. From your recent posts, I can only imagine the phone call âHi witch doctor, I think you are a quack that I am committed to prove is unsafe to practice medicine, could I come and shadow with youâ? click.... Might want to try a different approach and then we would love to hear about your experience! :)

Mona Morstein wrote: I have not published this, which I probably should, I agree. Technical writing is not my forte, I admit.

Tell you what ... I'm an ex-medical technologist AND a good technical writer. I also live in Phoenix, AZ so there should be no problem with logistics.

email me: abacaxi at hotmail.com and let's get this information out where it can do more people some good.

By Tsu Dho Nimh (not verified) on 15 Apr 2009 #permalink

Doug I haven't called you a witch doctor once yet.

... Because I have great respect for witch doctors.

At least THEY don't pretend to be real doctors.

In all seriousness, I actuallly CAN be charming, even to people I disagree with, and they appear to be reluctant even with my more CAM friendly class mates. Make of this information what you will.

Whitecoat Tales: First, based on your comments on this blog, my impression is that when you contacted an ND about shadowing him/her they could pick up on the fact that you are likely not doing so to "learn" but to ridicule and find flaw. Given that that is the likely situation, why would an ND agree to have you shadow him/her? And, I completely agree with the idea of finding out where the ND got his/her education. There are schools out there (Clayton College, Trinity College, etc.) that have accreditation, but the accreditation agency is not backed by the Secretary of Education. Furthermore, these schools are completely based upon online correspondence and do not offer or require clinical experience or patient contact of their graduates. So, "looks like a duck talks like a duck"? No, not exactly. I encourage the readers of this blog to look at the curriculum of say, Clayton College, versus that of colleges that have accreditation via CNME (an accreditation agency that IS backed by the Secretary of Education). There are 5 in the US and Canada, and the curriculum compares to that of conventional medical school.

Also, I wanted to point out that whereas you talk about the "significant evidence against the interventions suggested [...]" you fail to produce or cite any of this significant evidence. So far, one of the only people commenting in this forum who HAVE cited research has been Dr. Morstein.

Speaking of Dr. Morstein, yes, it would be nice for her to publish. That said, I'm guessing that the close-minded MDs in this forum would likely discredit whatever she would write, most likely because it would be in Case Review format, and therefore would not have a large population number. Fair enough, coming from a science and research background, I question those papers as well.

Point is, no matter what we say, no matter how well we articulate what we say, and no matter what "evidence" we show you, you will interpret it as you wish.

I have always found it interesting that throughout the history of science every time a person or group of people proposes an idea that goes against the currently accepted paradigm they are considered "crazy" and their idea or theory is rejected. And, almost inevitably, each time this occurs that "crazy" person's theory is accepted over time, and then we can't believe we ever thought otherwise (i.e. can you believe we once thought the earth was flat??). I just think it's funny that still, in this day in age, we continue that closed-minded cycle.

By Andrea LaCoss (not verified) on 15 Apr 2009 #permalink

@53 - "it's if what she sees in her office isn't real until she publishes... can't be real because you didn't learn about it in 'real' school.

It may be real, but unless she publishes, no one can know about it, and it can't be taught elsewhere. That's what scientific publication is really about: spreading the knowledge of what works and what doesn't work.

Ever hear of Ohm's law? It's called "Ohm's Law" and not "Cavendish's Law" because Ohm published his research and Cavendish didn't. In the late nineteenth century, long after his death, James Clerk Maxwell looked through Cavendish's papers and found things for which others had been given credit. Examples of what was included in Cavendish's discoveries or anticipations were "Richter's Law of Reciprocal Proportions", "Ohm's Law", "Dalton's Law of Partial Pressures", principles of electrical conductivity (including "Coulomb's Law"), and "Charles's Law of Gases".

By Tsu Dho Nimh (not verified) on 15 Apr 2009 #permalink

Tiffany Taylor said "What we really need is the funding for the research and publication of the existing evidence found in the thousands and thousands of patient charts in our offices and clinics across the country. I think that would lead immediately to more funding for more and better research, and for the extended residency education that most of us would welcome. Our medicine is very complicated, the protocols are far from standard, and the investigative research required to determine the causative factors in complicated cases can be enormous.

You're making to too hard. All you really need is a small study with decent incoming diagnosis and a reasonable follow-up time and good enough records to be able to decide what really happened.

Your office probably has enough patient records about a single condition to do this sort of study. Certainly enough to make a reasonable case series that may show that "whatever you did" is interesting enough to warrant further testing.

I'll make the same offer to you as I did to Mona Morstein: I'm an ex-medical technologist and a very good writer. You get the proper clearances from whatever agency is needed and I'll collate and write up the data. The only restriction is this: we do it with a before-the-project statement of hypothesis about the therapy of using X for condition Y and whatever the results are, positive or negative, they get published.

This is put up or shut up time ... I have the time and writing ability, you have the data.

By Tsu Dho Nimh (not verified) on 15 Apr 2009 #permalink

Tsu Dho Nimh: Love the idea! I canât speak for Dr. Morstein, but I highly doubt she will be back to this blog forum with the ignorant and arrogant disrespect that she received. If you are really concerned for the common good in the world then I would suggest contacting herself to make your proposal. She is only one of the brilliant doctors in our profession. I am sure a lot of NDs that are having the same success would take up your offer. A free offer right?

Whitecoat Tales: I appreciate what you just said. I want to personally invite you to come visit our medical clinic in Scottsdale, Arizona and shadow for a day or two. I am a poor struggling medical student with not much room, but you are more than welcome to crash at my place. Email me if you are ever interested: dcutler@scnm.edu

I donât represent our profession or school and I donât claim to have a vast knowledge about medicine. I am only passionate about naturopathic medicine and know it will continue to bless peopleâs lives even if it doesnât receive the recognition it deserves. Because that is what it medicine is all about right? Not for our egos, glory or acclaim. But for our patients that deserve true health and true hope.

Andrea:

So, "looks like a duck talks like a duck"?

That was me, so I'll respond.

I encourage the readers of this blog to look at the curriculum of say, Clayton College, versus that of colleges that have accreditation via CNME (an accreditation agency that IS backed by the Secretary of Education). There are 5 in the US and Canada, and the curriculum compares to that of conventional medical school.

Bastyr University in Washington State is listed as a CNME accredited school. Their curriculum includes things like 'Oriental Medicine', acupuncture, and homeopathy. These are pure woo, unsupported by any well conducted studies, and have no place in a traditional medical curriculum. BZZZZZT. Thank you for playing.

I have always found it interesting that throughout the history of science every time a person or group of people proposes an idea that goes against the currently accepted paradigm they are considered "crazy" and their idea or theory is rejected. And, almost inevitably, each time this occurs that "crazy" person's theory is accepted over time, and then we can't believe we ever thought otherwise (i.e. can you believe we once thought the earth was flat??).

Yes, many ideas that seemed initially improbable, blasphemous, or otherwise trend-bucking have been since accepted as true facts. Please note that this is not because public opinion suddenly shifted because people suddenly realized they were being too mean or closed minded. Nor is it because the proposers of these wacky ideas were extremely charming or persuasive. It is because the ideas themselves, and the data on which they were based, actually had merit. Experiments were repeatable; people the whole world round could test the theories for themselves and be convinced by their own findings, rather than taking the original proposal on faith. If you were to show me some actual evidence to support your faith in Naturopathy--sound, reproducible, statistically robust results indicating the existence of meridians, or the efficacy of acupuncture, relative to placebo, or whatever--my opinion of the treatment/phenomenon in question would adjust accordingly. That's how science works. However, the thing to remember when invoking the Galileo Gambit is that to compare your ideas to those of Galileo, it is not sufficient to be persecuted. You must also be right.

By Jennifer B. Ph… (not verified) on 15 Apr 2009 #permalink

Jennifer: This is precisely what I meant when I said that you will interpret things as you wish. Okay, so Bastyr University trains people in Homeopathy, Oriental Medicine, and Acupuncture. Did you fail to notice the other classes? The extensive Anatomy, Physiology, Neuroanatomy, Microbiology, Pathology, Histology, Embryology, ahem Pharmacology, Gastroenterology, Gynecology, shall I continue, or do you get the point?

My guess is also that you have no idea what Homeopathy, Oriental Medicine, and Acupuncture really are. If you did, you would understand the challenge of creating decent and accepted research on Homeopathy. Oh, I'm sure you're going to tear me up for that statement. But, here's the basic gist of the problem with Homeopathic research: the remedies prescribed to patients are very, VERY specific to that person. One person with Problem X may be prescribed Homeopathic remedy A based on their specific characteristic symptoms, while another person with Problem X may be prescribed Homeopathic remedy B based on their different characteristic symptoms. So, you can't create a good trial of, say, Homeopathy being effective in treating nausea, because each person will be treated differently based upon their characteristic symptoms. Now, I don't really expect you to understand what I just said without actually having any idea of what Homeopathy is, but perhaps if you want you should go do a little reading on Homeopathy and then you will get it.

As for Acupuncture (and thus Oriental Medicine, since that is what Acupuncture is based upon...which you would know if you knew anything about the things you are condemning), there is actually quite a bit of research out there showing it's effectiveness. Furthermore, I would like to point out that MDConsult actually lists acupuncture as a valid treatment for Carpal Tunnel Syndrome (they also list hydrotherapy and supplementation with Vitamins B2 and B6 for this condition, but we'll stick to the Acupuncture topic).

I just ran a quick search on PubMed, and this is what I found:

"The Effects of Hand Acupuncture Therapy on Pain, ROM, ADL and Depression among Elders with Low Back pain and Knee Joint Pain."
http://www.ncbi.nlm.nih.gov/pubmed/19265308?ordinalpos=7&itool=EntrezSy…
_DefaultReportPanel.Pubmed_RVDocSum

"Moxibustion and other acupuncture point stimulation methods to treat breach presentation: a systematic review of clinical trials"
http://www.ncbi.nlm.nih.gov/pubmed/19245719?ordinalpos=8&itool=EntrezSy…
_DefaultReportPanel.Pubmed_RVDocSum

Of course, there were about 950 other articles, but it's late and I don't have the time to go through all of it.

Did you ever stop to think about why people come to see Naturopathic Doctors, and continue to come to see them, if the medicine, as you say, is "pure woo"? If the medicine was "pure woo" then it wouldn't be increasing in popularity. If the medicine was "pure woo" Naturopathic Doctors wouldn't be able to make a living, more and more states wouldn't be licensing for them, and alternative health care clinics wouldn't be added to existing conventional hospitals. Like my boyfriend's MD father once told me, "If acupuncture didn't work, it wouldn't have been around for thousands of years."

Ignorance is not bliss, my dear. Go educate yourself on things that you condemn and perhaps even go get an acupuncture treatment - I promise you won't be disappointed.

By Andrea LaCoss (not verified) on 15 Apr 2009 #permalink

Well Marc, it's a good thing that this is the United States and not Britain, isn't it? Especially since our Naturopathic Medical Schools are NOT run by Scientologists. Your logic is flawed.

By Andrea LaCoss (not verified) on 15 Apr 2009 #permalink

Andrea,

Homeopathic remedies are so dilute no active ingredient is present. Consequently, if these remedies have any biological effects, it won't be just the doctors re-writing their textbooks. The chemists and physicists will need to revise their textbooks also, particularly with respect to thermodynamics.

One must demonstrated that a homeopathic remedy can in some manner be distinguished from mere water before any clinical trials is warranted.

Clinical trials are not appropriate where the basic science is implausible. Under such conditions, every positive outcome will most likely be a false positive and will send us all on an expensive, wild goose chase for many years.

Settle the basic science first. Prove the remedy is more than water.

Do you MD's ever consider the origin, cause of disease before people have markers, or values, or symptoms?Yes we do.

This "cause vs. symptoms" argument comes up a lot from the alt med team. It appears that the word "cause" many not mean the same thing over there on the alt side. The alt notion of disease-cause seems to involve some alleged imbalance of the vital force.

Admittedly, we don't talk about the vital force on the science-based side.

@Tsu Dho Nihm-- Damn! Poor ol' Cavendish! He really got the shaft. He's like the Emily Dickinson of Physics!!!

To test Earth's density it takes a torsion balance and one gee,
One torsion balance, and a gee,
For gravity.
The gravity alone will do,
If lead spheres are few.

My guess is also that you have no idea what Homeopathy, Oriental Medicine, and Acupuncture really are. If you did, you would understand the challenge of creating decent and accepted research on Homeopathy. Oh, I'm sure you're going to tear me up for that statement. But, here's the basic gist of the problem with Homeopathic research: the remedies prescribed to patients are very, VERY specific to that person. One person with Problem X may be prescribed Homeopathic remedy A based on their specific characteristic symptoms, while another person with Problem X may be prescribed Homeopathic remedy B based on their different characteristic symptoms. So, you can't create a good trial of, say, Homeopathy being effective in treating nausea, because each person will be treated differently based upon their characteristic symptoms. Now, I don't really expect you to understand what I just said without actually having any idea of what Homeopathy is, but perhaps if you want you should go do a little reading on Homeopathy and then you will get it.

So, what makes the difference in the individual treatment? Since it's not the water, which can be shown to NOT contain any reagent (ok, you have a one in million chance your D10 still has a single molecule), is it the the frequency and direction of the succussion? Or is the hand in which it is held during the succussion the defining factor, or maybe the gender of the preparer, machine succussed vs. hand?

Andrea LaCoss said "If you did, you would understand the challenge of creating decent and accepted research on Homeopathy. Oh, I'm sure you're going to tear me up for that statement. But, here's the basic gist of the problem with Homeopathic research: the remedies prescribed to patients are very, VERY specific to that person."

A couple of years ago Dr. Steven Novella was part of a panel discussion/debate on homeopathy in Connecticut. One particular homeopathy, Andre Saine, made the very bold (and testable) claim that homeopathy works better for rabies, see:
http://www.theness.com/neurologicablog/?p=41

Since I watched the video and read that blog posting I have been asking where the actual data is that shows homeopathy is better for rabies. Do you have it? (by the way, it needs to available at my local medical school library)

Holy fucking shit. Ok...

Bastyr University trains people in Homeopathy, Oriental Medicine, and Acupuncture. Did you fail to notice the other classes?

Andrea, my point is it doesn't matter what other classes they teach. If the theory and practice of these pseudosciences are part of the curriculum, it's tainted. I'm not going to be impressed by someone who can describe the paths of my cranial nerves in detail if they're going to be instructing me on vitalism in the same breath.

I haven't had enough sleep or enough coffee to tackle the rest of your logical fallacy-fest. At some point I definitely need to write a CAM version of the Courtier's Reply, though, because comments like yours just scream out for such a rebuttal. Cheers.

By Jennifer B. Ph… (not verified) on 16 Apr 2009 #permalink

Dr. Benway: I completely respect your argument that some higher dilutions (and thus potencies) of Homeopathic remedies have no traceable amount of the original substance. This is a property of Homeopathy that the greatest Homeopathists have been trying to explain for many years. I have heard one explanation that the energy of the original substance remains. This, of course, does not a rebuttal make. So, instead, I would like to point out a basic fundamental principle of the philosophy of science: truth and fact in science is only truth and fact until we discover new ways of testing. So, okay, under our current testing capabilities, Homeopathic remedies have no traceable amount of the original substance. But, perhaps when there is a new way of testing, we will determine specifically what about the remedies makes such substantial effects.

I will be the first to admit that it took me a very long time to accept Homeopathy. And, your above point was definitely in my mind. I couldn't explain it, scientists couldn't explain it, professors couldn't explain it. So why did I eventually accept Homeopathy? Because I saw it WORK, time and time again, both in myself and friends, but also in our patients. I know, I know, that isn't enough for you:)

In regards to your comment about Naturopathic medicine having a different idea of what "cause" is. In my 3 years of training thus far, though I have been taught about the vital force, I have not heard it used in context of being a "cause" of any disease. Rather, we just take a few steps back and find the cause. For example, I had a peds patient who was diagnosed by an MD with Generalized Anxiety Disorder and Sensory Disorder. She had extreme sensitivity to tactile stimulation, such that putting on clothing was the worst part of her day, causing great anxiety, temper tantrums, etc. She was also sensitive to auditory and olfactory stimulation. I was baffled as to what to do for her, but thankfully by attending physician was not. He had apparently seen this before, and seen this diagnosis, and said that many times these kids have a candida issue and that we should run an OAT test. I was skeptical, but hey, I"m just a 3rd year medical student. Results came in, she had candida. She was put on diflucan (I think, I may be wrong about the drug given), and within a week the mother was calling the office in TEARS because her daughter was doing so much better and no longer had the extreme issues with the clothing that she once did (a huge stressor in the family), noise, or smells. THAT'S what I mean about finding the "cause".

By Andrea LaCoss (not verified) on 16 Apr 2009 #permalink

I've been staying away from this very interesting discussion, but really, any defense of magic water is destined to be pure idiocy. You cannot (or more properly should not) assume homeopathy works, and then try to find "new ways of testing" until you've validating your pre-ordained conclusion. It's not how the universe works.

Don't waste your time on charlatans like PalMD who graduated from a school that teaches non-science based medicine, pseudo-scientific, deceptive, and immoral health practices, and Phara shill voodoo. How is this so? Because Rush University Medical School received the same Federal Accreditation from the Higher Learning Commission, a Commission of North Central Association of Colleges and Schools as the Southwest College of Naturopathic Medicine. Oh yeah, other colleges with this dubious accreditation are the Rosalind Franklin University of Medicine and Science, Midwestern University, University of Illinois, Loyola University, Southern Illinois University, Indiana University, Notre Dame, Medical College of Wisconsin, Mayo Clinic College of Medicine, and many others. Beware of their theology-driven scientistic denialism! A pox on those who dare to defy the only true believers in drugs-and-surgery-cures-all! If it isn't "proven" by a PHRMA funded DBPCRCT costing hundreds of millions of dollars and resulting in a 20 year patent, then it doesn't fit the model and is blasphemy until it does!

And if you are wondering, I practiced allopathic medicine with the school-indoctrinated blinders on for over 25 years. It has its place, but is inherently limited. I practice with MD's, DO's, and STATE LICENSED ND/NMD's who put their patients welfare above their scientistic (NOT SCIENTIFIC - THIS IS ANYTHING BUT SCIENTIFIC) theology. It's been hilarious reading your lame empty "challenge" responses, including refuting your own studies and recharacterizing the results when they don't fit your theology. At least the new AMSA medical school students are getting it, like the MD/DO's at ACAM, A4M, and numerous other organizations. Dr. Bill Benda MD was right when he said knowledge in the MD community take 2 generations to take hold - it requires the passing of both the teachers and their students... keep up the comedy!

Speaking of pseudoscience, what do you all think of Physics? Because, by all accounts and definitions of "pseudoscience", much of what Physics teaches us about the world is purely based on currently untestable theory. Dark matter? It's just a guess. String Theory? An obsession of many, yet unproven. Everything past the event horizon? Theories, but we can't test it yet. Haha...and PalMD...I'd like to point out that we don't entirely "know" how the universe works, as large chunks of our understanding are based on theory and "pseudoscience". :P

Point is, don't close your mind off to something just because we can't quite test it yet. The more open-minded we are (to all scientific fields), the better the progression of science and the human race.

By Andrea LaCoss (not verified) on 16 Apr 2009 #permalink

@DrRx: Whiskey. Tango. Foxtrot.

@Andrea: there is a difference between knowing how the universe works (essentially) and knowing every detail. While we have not yet isolated a "graviton", we still can measure and predict the force of gravity. Also, gravity never fails when it is predicted to work (no one falls off the earth).

To be "open-minded" to ideas that clearly violate all of our working ideas of physics and chemistry is either daft, arrogant, or both.

PalMD your god ego is coming through just a little bit. But don't fret, there is actually a homeopathic remedy to restore you back to sane health.

But in the meantime, please teach us how else the universe works.

Yes, many topics in physics are currently untestable and may remain forever untestable, given the distance or size of the object. However, we don't design rockets according to string theory. We use tested and testable physics for our real world applications. And if string theory is ever verified, and if using it will affect how we design rockets, you can bet that it will be incorporated into rocket design.

Thanks palMD for this thread. Its devolution to... whatever it turned into has been fun reading :) I'm particularly fond of someone conflating magic water with theoretical physics.

Anyhow, no one ever answered me at #27, or I guess more specifically didn't call out the two controversial tests.

Dacks, another homeopathic remedy would fit you as well.

But please continue on teaching us about rocket designs.

We have an omniscient god that is teaching us about how the universe works and a rocket scientist that is teaching us about rocket designs. How amazing it is on how we can become anything we want on the interwebs.

Anyone else need a homeopathic remedy?

And also check out my latest blog that will be coming soon and I will teach everyone on how they can fly.

By Doug aka Superman (not verified) on 16 Apr 2009 #permalink

refuting your own studies and recharacterizing the results

It appears you are among the many Doctors who failed journal-reading 101

But it's always interesting to see the Woo Converts denounce all that they don't understand as "also religion - beacuse I never figured out the science"

Further kudos are in order for your successful Pharma Shill gambit. It's impressive when you try to foist that on multiple people who have no financial stake in the process.

Because, by all accounts and definitions of "pseudoscience", much of what Physics teaches us about the world is purely based on currently untestable theory.

The difference is, the physicists aren't making decisions to decide if someone lives or dies based on currently untestable theory. So when you're prescribing that herbal remedy for refractory asthma that hasn't been proven to work instead of putting him on inhaled corticosteroids, long acting beta agonists and making sure he has a rescue inhaler, ask yourself what a theorhetical physicist would do in your position.

don't close your mind off to something just because we can't quite test it yet. The more open-minded we are (to all scientific fields),
(emphasis added)

I agree. Scientific fields. And when you start making testable hypotheses that are consistent with basic science, testing them, and they are accurate, we'll talk.

Until then, your voodoo has no place in medical school.

You're forgetting that we are not only trained in "voodoo" but are trained in how to APPROPRIATELY diagnose and treat disease. If we see an emergency situation, we refer out. If we see a situation that cannot be dealt appropriately with "natural" therapies, we prescribe (or refer, depending on the situation). That's the beauty of attending an accredited Naturopathic Medical school versus a foo-foo Naturopathic school (see above in my description of the difference)...we're not just trained on our side, we're trained on your side too. By learning both, we can understand the limitations of the medicine, whether natural or pharmacological.

And BTW, what is a Pharma Shill?

By Andrea LaCoss (not verified) on 16 Apr 2009 #permalink

we're not just trained on our side, we're trained on your side too.

Andrea, the point you are (I suspect willfully) continuing to miss is that your 'training' on 'our side' is woefully insufficient--even if it weren't diluted with woo, it would be so far below the standard of expertise of someone trained as an MD that it's a joke to even compare the two. This was, I think, the most informative outcome of PAL's little Naturopathy Invitational. Your medical training is insufficient to adequately assess and treat something as complex as a human body. Period.

And, since you asked: pharma shill

By Jennifer B. Ph… (not verified) on 16 Apr 2009 #permalink

To state the good ideas one embraces is not enough. One must explain what is rejected and why. Science is not a method for accepting what is true. It's a method for rejecting what is false.

So what therapies, dear naturopaths, do you reject? What standard do you use to reject those ideas?

Disclaimer and anecdote: Our extended family includes a person who was hospitalized in the county psyche ward after an incident. When she was released six weeks later she was doing very well. She had been diagnosed with a bipolar disorder, given some medication and talk therapy.

Unfortunately when she was released there was no follow-up, so she went back to her Bastyr trained naturopath. The naturopath gave her homeopathic meds to take instead of the real medication.

It did not work.

Even our relative realized it, she wrote a long letter to her parents explaining that homeopathy does not work. When they told us that at a holiday gathering I said I could have told them that! Anyway, fortunately she went back to real doctors and is doing much better.

Oh, funny thing about the Pharma Shill bit. This relative used to complain on how expensive the "prescriptions" the naturapath had her get at a compounding pharmacy. This includes the homeopathic stuff. I believe she no longer sees the naturapath!

Anyway, still waiting for the evidence that supports Andre Saine's assertion that homeopathy works better for rabies.

Andrea,
You said

You're forgetting that we are not only trained in "voodoo" but are trained in how to APPROPRIATELY diagnose and treat disease. If we see an emergency situation, we refer out.

Firstly, you are not trained in how to appropriately diagnose and treat disease, as clearly indiciated by palMDs entire article series here. In MD school, we spend 4 years plus a residency to appropriately diagnose and treat disease.

You are suggesting in that in 4 years without hospital work you can do the same, while also appropriately prescribing your "CAM". It is unlikely in the extreme that all of the people in naturopathic medicine learn more than twice as fast and twice as much as MD students, so it would seem something must be sacrificed. Unless you guys have revolutionary secret study methods too!

Secondly, I can't find an official guideline for naturopathic medicine, but what I can find shows that Ns are woefully ill-equipped to know what "natural therapies" CAN'T treat!

Emily Kane, ND, graduate of Bastyr, wrote an article published by the AANP.
http://www.healthy.net/scr/Article.asp?Id=783&xcntr=3

She says

For an acute attack:
Cramp bark 7.5 mL of tincture
Ephedra 5 mL
Skunk cabbage 5 mL
Thorn apple 3-5 mL
Lobelia 7.5 mL
Cayenne pepper 5 mL
Mix these ingredients together and take 2-3 dropper fulls every 20-30 minutes until the attack subsides.

Do you repudiate her suggestion of a dangerous, ineffective home remedy for acute asthma attack? Because treating an asthma attack this way for an MD would be malpractice.

She also recommends digestive enzymes as supplementation because

digestive enzymes can help reduce mucus formation and prevent undigested food from "leaking" into the blood stream, which would cause a systemic inflammatory reaction.

This is complete bilge. Undigested food "leaking" into the blood stream? Yet you claim that Ns learn anatomy and physiology...

The entire article is chock full of useless therapies or marginally useful home remedies and mythical treatments known not to work. I don't care how much anatomy and physiology you take. If you tell an asthmatic to use these therapies, you can't be a doctor of any sort.

...we're not just trained on our side, we're trained on your side too.

To reiterate my point, because it's often misunderstood:

If your science is sound, it ought to screen out illogical or unsupported claims. Therefore, a single failure on that count proves your science inadequate. Doesn't matter how much good information you accept. Doesn't matter how many classes you take.

Science rejects.

Doug, I wasn't claiming to be a rocket scientist (hey, is that a cliche?), merely exploring the weakness of an argument that equates homeopathy with string theory. (Hint: you use unsubstantiated homeopathic remedies, rocket scientists don't use string theory to build rockets.)

Which profession has double the suicide rate of the general population? Has a life expectancy 7 years less than average? A substance abuse problem equal to those in the lowest socioeconomic class? A job satisfaction rate lower than that of slaughterhouse workers?
The almighty MD. Really, how well is the allopathic medical model treating those who know it best?

http://emedicine.medscape.com/article/806779-overview
http://www.ncbi.nlm.nih.gov/pubmed/15569903
South Med J. 2000 Oct;93(10):966-73.
The painful truth: physicians are not invincible.
Acad Med. 2009 Feb;84(2):269-77.
An exploratory study of resident burnout and wellness.
Physician Burnout and Stress Now Reaching Critical Levels
http://www.emediawire.com/releases/2007/4/emw516822.htm

By PA turned ND (not verified) on 16 Apr 2009 #permalink

Dr. Benway, you write "If your science is sound, it ought to screen out illogical or unsupported claims. Therefore, a single failure on that count proves your science inadequate. Doesn't matter how much good information you accept. Doesn't matter how many classes you take."

but as per the video posted earlier, which I'll re-post just in case you missed it,
http://video.google.com/videoplay?docid=-7758662442132419447&hl=en

if this is the "sound science" you are talking about, I'm not sure I want to practice that medicine because it doesn't seem like that science has our patients' health in mind. Also, if this is the medicine that your and PalMD's ego has been derived, well then I hope it serves you well with your patients. Then again, I'm sure this is the only case where the studies are tainted with a little conflict of interest.

By Brent Cameron (not verified) on 16 Apr 2009 #permalink

My ego, my arrogance, my asshattery---these are all irrelevant ad hominem arguments. My personality is irrelevant to science. If you can't reason that out, then you'll probably have to add offensively insulting to my labels.

The scientific method aka "reason" aka "methodological naturalism" involves

1. Corroboration
2. Falsification
3. Logic
4. Parsimony

Claims about the world may be falsified and thus rejected.

This is not my particular version of science. This is it.

Speaking of pseudoscience, what do you all think of Physics? Because, by all accounts and definitions of "pseudoscience", much of what Physics teaches us about the world is purely based on currently untestable theory.

Wrong. The current frontier of physics deals with as-yet-untestable ideas. That's what being cutting-edge is all about. If it were easy, we'd be done by now. However, not all the lessons of physics lie on the edge; far from it. Indeed, the very reason those novel concepts haven't been tested yet is because the circumstances in which they are directly relevant are far removed from everyday experience. By that same token, they cannot constitute the bulk of what we know about the world.

Dark matter? It's just a guess.

All scientific discoveries begin as guesses. The guesses which work survive to be granted tentative acceptance. Dark matter is a guess which works. Dark matter has been observed: it has detectable effects on galaxies and intergalactic gas clouds.

String Theory? An obsession of many, yet unproven.

Science does not deal in proof. To speak of a hypothesis being "proven" or "unproven" reveals a sloppy understanding of the way science operates. We raise or lower our confidence in hypotheses based on the collected evidence of empirical observations. "Proof" is left to the mathematicians and the moonshiners.

And, again, if it were easy, we'd be done by now. There will always be an edge, the frontier between the known and the unknown. Sometimes, theory will lag behind experiment, and on other occasions, experiments will be too difficult to perform, and theorists will be working blind. In either case, the challenge of the unknown will draw those with the honest curiosity to explore. The current situation is exactly what one would expect from a healthy, active field of inquiry.

"@Andrea: there is a difference between knowing how the universe works (essentially) and knowing every detail. While we have not yet isolated a "graviton", we still can measure and predict the force of gravity. Also, gravity never fails when it is predicted to work (no one falls off the earth)."

PalMD, not to point out the obvious but you just contradicted yourself with that statement. Andrea referenced dark matter which is something that describes the exact opposite of what you just said about Newtonian gravitational effects of things we can observe and understand. The hypothetical concept of "dark matter" came about because gravity wasn't acting entirely as it should based on our understand that you say "never fails." So we theorize something else is present, a "dark matter" that must exist to cause this abnormality and tie things back together in our heads logically.

If you don't like Naturopathic medicine that is fine especially as a person who deals in science like yourself. But to be effective in the advancements of science you really should take things as simple as the scientific method into consideration when making attempts to gain understanding and debate things. That involves stepping back from yourself, really learning what they do and *then* postulating your position. That is science 101 and from what I've seen you do in this "challenge" I do not see any objective and unbiased thinking from your end. You've already decided what you believe before you entered the experiment! So what is this, an attempt to smear others and create debate based on your already perceived reality? I hate to break it to you but that says something somewhat important to me. It shows me who won't be making any medical breakthroughs anytime soon kicking back with that boxed in know-it-all mentality.

I'm no MD or ND but I am in engineering and technology. Like medical science or all sciences in general my own field changes overnight where we have to learn to cope with that or risk falling behind. You cope by really learning the intricacies of your target methods. If I had a dollar for every time an accepted scientific "reality" has changed in my lifetime I would be a millionaire by now. That in mind, I remain open to things even when I think I know the answers. Way too many times have I seen my own preconceived concepts turn right back around on me. To be blunt, I don't think your form of medicine or their form of medicine are entirely correct. I think truth usually sits somewhere in between perceived realities.

Before becoming an ND, I became an Anthropologist. One of the first courses I took during my undergrad years was a course on the subject of Medical Anthropology. This course addressed the Western Biomedical model as a cultural creation that we have exported to other parts of the world. Along with this model (that came mostly out of Europe and specifically England during the late 18th century) also emerged the idea of racial and cultural superiority (this is why uneducated MD's can say, with complete confidence, that any form of medicine that does not fit the Western Bio-Medical Model is Snake oil. And that their medicine is superior to that of the Chinese medical model or the Ayurvedic model developed in India over centuries). You see, this attitude is the attitude of the bigot.
Often this bigotry and arrogance has had catastrophic results, Vioxx, Celebrex, Thalidomide, HRT to name a just few. Rest assured that, those tens of thousands of deaths were not caused by ND's, but MD's alone. Yes, MD's! You alone can take full credit for those deaths; and countless others due to iatrogenic causes. Indeed, it would take us centuries to kill as many people as MD's have killed due to misplaced confidence in drugs, over prescribing, and faulty research.
I wont bore you every atrocity instigated against indigenous people in the name of the "good biomedical" model at the cost of ignoring the local voice. Their are far too many horror stories. From forced sterilizations to poor nutritional management of reservation populations (where the average life expectancy is often less then 40 years sometimes), to the forced adopting out of native children to white families (with the "scientific belief", that they could be taught to be "white", and thus superior).
What I would suggest is that MD's began to research other forms of medicine and begin to be a little more self critical of there own brand of medicine and their own bloody history before they do any more damage.

That sums it up for me because I know there are wise MDs and wise student MDs that have a desire to work side by side with NDs for the ultimate benefit of the patients. It is happening now and will continue to happen.

So I can begin to forgive the history of orthodox medicine and the major transgressions committed against all humanity.

Those that blatantly continue in that same path of transgression will be held accountable. Maybe not in this life, but in the life to come!

Oh wait, what? Forgiveness, spirituality, spiritual health and the after-life on this âscientificâ blog? âHow dare he! We are the omniscient gods!â

Alrighty, I asked for it so bring all your âscientificâ clinical studies for you own distorted agenda, your apostate philosophies made of men, your witty labels, or your fervent charges of âbeing holier than thouâ. I still can forgive you.

But never preach to me that you know what TRUE science is! Naturopathic medicine is only a very small part of the bigger picture of truth.

By Doug Cutler (not verified) on 16 Apr 2009 #permalink

I like the very first comment left on this site:

"I appreciate your blogpost, as it opens up the possibility for a real exchange"

This blog has been anything but. The comments left have been nothing but a population of close-minded people on one side and shunned âvoodoo-istsâ on the other. I guess that is not too surprising that this is what it would turn out to be when the top of this pages characterizes this topic to be about âmodern shamanismâ and âabsurd medical claimsâ. Rather comical at the worst and tragic at best to see two sides of what should be complementary fields arguing over frank superiority on one side and the right to legitimacy on the other. After reading through every comment up to this point I am not only ashamed with a select few M.D.âs and ârealâ medical students, but also to inevitably be forced into their company by my own status as an OMSII, surely you arenât shunning D.O.âs too know are you.

I also find it rather amusing you challenging NDâs to a âpatient-offâ.

This whole blog is filled with nothing but over-opinionated people running their mouth. May I offer a few examples.

Dr. Benwayâs description of the âGotchaâ in comment 13. âWelcome to med school and residency. Welcome to the practice of medicine, the land of the gotcha.â As if they were more of a prolonged trick rather than a teaching environment. Is that what this is PalMD? A teaching environment? Are you teaching us what you know about medicine and NDâs? Are you âpimpingâ us? Sounds like you need to go back to Day One when they taught you that helping the patient was just as important as a proper diagnosis. If you are trying to honestly learn about Naturopathic Medicine, if anyone is by reading this, you are currently doing a poor job. Wikipedia is a more reliable source of info than this blog.

Or later Dr. Benway you say that, âMedical schools are accredited by the Association of American Medical Colleges, which has not accredited any school of naturopathy.â Schools of Naturopathy, you are right, are not accredited by them because it is not their governing/accrediting body. By itâs own website (http://www.aamc.org/medicalschools.htm) the AAMC describes its duties thus: âThe AAMC represents the 130 accredited M.D.-granting U.S. medical schools and the 17 accredited Canadian medical schools.â It does not say that it is licensed or responsible or anything about being suited or responsible or even necessary for accreditation of other medical schools. Would you be happier if Proud ND had stated âN.D.-granting U.S. medical schoolâ? Because it certainly is that. Naturopathic medical schools donât need the AAMC to be called a medical school. The AAMC only represents M.D. granting schools by its own admission. You seem to be caught up in the grammar. How about this: fully-accredited 4 year Naturopathic Medical School? Better for you? The fact is that they donât need the AAMC because they have the AANMC which recognizes the following accrediting institutions: The US Department of Education, The Council on Naturopathic Medical Education (CNME), and the North American Board of Naturopathic Examiners (NABNE). The AAMC doesnât accredit them for the same reason they donât accredit you. Pretty simple. Or for the same reason the AACOM doesnât accredit you. You certainly donât accredit them so are Osteopathic medical schools not really medical schools now either because they arenât accredited by the AAMC? Do you not approve of the AACOMâs credentials?

Or Whitecoat Talesâ thorough knowledge of all things medicine (being a 3rd year MD student, wow!) when he states in comment 26 that âIf you want to be a pcp take boards and do a residency.â How about this: if you want to comment, do your homework first. (Thatâs a quote from me by the way). The truth is they do. Both of those things. The NABNE gives the board exams and you could find Naturopathic residencies if you tried. Iâll get this one out of the way for you. But their test isnât the USMLE so therefore itâs no good and shouldnât be considered as valid. The COMLEX isnât the USMLE either. Is it not valid? Have you, Whitecoat Tales, never seen a D.O.? (P.S. thatâs what the COMLEX is for, in case you didnât know).

In comment 33 PalMD says that he is unimpressed, presumably by Dr. Morsteinâs description of her typical first-time visit with such a patient. What exactly are you unimpressed by? The fact that she responded mainly to the diabetes/hypertension/weight/LDL/coronary heart disease challenge and did not tell you explicitly what she would do about his LAD stent?

Or when PalMD responds to Dr. Morstein by saying that âthere is also reasonably good evidence that aiming for the goal has benefit in and of itself.â Are you implying that Dr. Morstein is not aimed at this goal? Sounds like she is. So by your own admission of the inherent beneficence of aiming, magically there is now no benefit in her treatment regimen of âtwigsâ? Since when are vitamins (recommended by the FDA), fish oils (recommended by the American Heart Association, http://www.americanheart.org/presenter.jhtml?identifier=4632, you are still willing to admit weâre talking about his heart here Iâm assuming), etc. âtwigsâ? Is that what they taught you in Pharmacology 101 (since it was so adequately recommended by âInternistâ) or Biochemistry? That things besides Sulfonyureas (Glyburide) and Biguanides (Metformin, just for you Internist, I took Pharm also) are twigs? Since when do vitamins and fish oils and these other âtwigsâ commonly cause lactic acidosis like Biguanides? When were omega 3 fatty acids and Vitamin D taken off the market like Phenformin and Buformin were?

Or in #49 in response to Dr. Jen. âYou don't form a conclusion, act on it, and then hope for data. Science---youâre doing it wrong.â Forming a conclusion (say, that a treatment is effective), acting on it (say, giving that treatment) and then hoping for data; sounds kind of like a Phase II clinical trial. Wait, they do that with drugs, donât they? Pharmaceuticals arenât science-based now?
âThats a whole lotta biochemistry, got any clinical trials? Extraordinary claims demand extraordinary proof.â This one was good too. Whitcoat Tales, are you honestly asking for clinical trials on basic Biochemistry? I know it was two years ago since you opened one of those books but what more do you need? How about a refresher. Start on page one if you have to.

Or (I skipped a few) Jennifer B. Phillipsâ response to Andrea: âAndrea, the point you are (I suspect willfully) continuing to miss is that your 'training' on 'our side' is woefully insufficient--even if it weren't diluted with woo, it would be so far below the standard of expertise of someone trained as an MD that it's a joke to even compare the two. [â¦] Your medical training is insufficient to adequately assess and treat something as complex as a human body. Period.â Really? You donât think they could diagnose someone with Erbâs palsy (that seems kind of complicated enough beings as it could be other things like a C5/C6 radiculopathy or other things too complicated for anyone but an MD to know, right?) Jennifer show me a breakdown of their credit hours of Anatomy / Biochem / Pharm / Micro / Neuroscience / Physiology / Path, etc. that proves their training is inadequate. Show us all how their courses in Endocrinology, Obstetrics, Minor Surgery, and so on arenât up to par. And being watered down? Iâm an Osteopathic medical student and we learn OMM. If you donât know what that is, look it up. Does that water us down enough that D.O.âs are incompetent? Hardly. In fact Iâve taken nearly 20 credit hours of âwateringâ in things like HVLA, lymphatic techniques, muscle energy, counterstrain, Chapmanâs points, etc. Donât know too much about them? Thatâs fine, read the JAOA. http://www.jaoa.org/contents-by-date.0.shtml. Have you even heard of Chapmanâs points? Does that mean itâs not legit? Hardly. I invite you all to do research on it. Itâs out there. Peer-reviewed and all. And you know whatâs even funnier? Most of these neurolymphatic reflex points line up with acupuncture points. Acupuncture is woo? Do your own research before you keep yammering.

All that is going on here is that a small group of people are taking pot-shots. Like PalMD calling Caitlin naïve and regarding her as dangerous because she is âbucking those in your profession who have been at this longer.â Whitecoat Tales, the glorious 3rd year student that he is, is bucking the same way at Dr. Morstein or Dr. Jen or any other licensed N.D. on this forum. Newsflash. The rank ignorance in this forum is stifling. The three major offenders in this blog, I think you know by know who you are, are stuck in an era of medicine that still thinks the world is flat or that Vioxx is really great. Donât get what I mean? Ask PalMD and Dr. Benway, they seem to have all the answers for everything; except your original case. Throw your answer at us. Youâre interested in âpeer-reviewedâ science right? Let us try to shot some holes in it and call you stupid for a change.

And for those of you interested in science, here is some science for you PalMD and the rest, straight from Dr. Benway.

1.Corroboration â I all think you are all full of yourselves; I just did in the lengthy text preceding this
2.Falsification â too hard to dispute that fact
3.Logic â reread your posts
4.Parsimony â thatâs as less complex as I can get

Or rather, I observe that you are full of yourselves, I admit this as my hypothesis, I predict that others will come to the same conclusion, and what for the data to pour inâ¦

By Matthew White (not verified) on 17 Apr 2009 #permalink

also emerged the idea of racial and cultural superiority (this is why uneducated MD's can say, with complete confidence, that any form of medicine that does not fit the Western Bio-Medical Model is Snake oil. And that their medicine is superior to that of the Chinese medical model or the Ayurvedic model developed in India over centuries).

Ha! I knew it was coming! Anytime someone talks about science, it doesn't take long before some of the psuedoscientists breaks out the race card.

I'm an easterner (Indian), not a westerner, so I'm immune to your "western biomedical model is a racist social construct" gibberish! I can tell you from experience that Ayurvedic medicine is (mostly) snakeoil and the only reason I'm not saying it's all snake oil is because some parts of it haven't been investigated yet. The beautiful thing about the scientific method is that it gives us a way to separate anything that may end up worthwhile from that which is gibberish.

This attitude is NOT the attitude of the bigot. It is the attitute of the reasonable man - extraordinary claims demand extraordinary evidence.

Often this bigotry and arrogance has had catastrophic results, Vioxx, Celebrex, Thalidomide, HRT to name a just few. Rest assured that, those tens of thousands of deaths were not caused by ND's, but MD's alone. Yes, MD's! You alone can take full credit for those deaths; and countless others due to iatrogenic causes. Indeed, it would take us centuries to kill as many people as MD's have killed due to misplaced confidence in drugs, over prescribing, and faulty research.

Disappointing. Noone claims doctors make no mistakes. But we have a system to filter out those mistakes.
None of the naturopaths has even suggested an excuse for, or repudiated the Emily Kane AANP article suggesting dangerous unethical treatment for asthma NOW, you judge my profession for mistakes without acknoledging your own.

You also are making an ethical mistake when you discuss iatrogenic illness. Is there a difference between knowingly letting 100 people die, and making a decision that will actively kill 10 but save 90? Surely the second is the ethical choice, though pseudoscience advocates will skewer us over any deaths.
Yet your profession chooses therapies that have similar risks, without similar benefits, as detailed elesewhere on these comments.

I wont bore you every atrocity instigated against indigenous people in the name of the "good biomedical" model at the cost of ignoring the local voice. Their are far too many horror stories. From forced sterilizations to poor nutritional management of reservation populations (where the average life expectancy is often less then 40 years sometimes), to the forced adopting out of native children to white families (with the "scientific belief", that they could be taught to be "white", and thus superior).

This entire tirade has nothing to do with medicine. You're conflating western civilization and western medicine.

What I would suggest is that MD's began to research other forms of medicine and begin to be a little more self critical of there own brand of medicine and their own bloody history before they do any more damage.

I suggest you investigate your bloody present before criticizing my bloody past.

This course addressed the Western Biomedical model as a cultural creation that we have exported to other parts of the world.

This is where postmodernism --which I don't bash generally-- is taken too far.

Science, like the rules of arithmetic or Euclidean geometry, is not political advocacy. It's not feminism or liberalism or fascism.

"Science" or "reason" or "methodological naturalism" cannot be merely one of many possible narratives. For if that were the case, then t[o80 uqu0[ jgri94 g 90 jgapq jgcls. In other words, scholarship or rational debate itself become meaningless once the rules of debate are deemed arbitrary.

Science is a method for assigning confidence in the truth of various claims about the world using four sorts of tests:

1. Corroboration: Claims that can be independently corroborated deserve more confidence than claims that cannot be corroborated.

2. Falsification: A claim is deemed "unproven" until it's been subjected to some effort to prove it false. We are under no obligation to take unproven claims seriously.

3. Logic: Claims can be rejected if they are self-contradictory, or if their necessary implications are self-contradictory, or if they lead to a contradiction involving well established facts without somehow resolving that apparent contradiction.

4. Parsimony: Claims with the fewest unfounded assumptions are preferred.

Using these four tests we can create a shared map of reality, no matter our history or culture.

The corroboration rule means that I extend to you the right to double-check what I say. This is the opposite of cultural imperialism, or imperialism of any sort.

Without science, without a mutually agreed upon method for separating facts from supposition, there can be no love. At best, two lovers might be near each other while masturbating to their own private illusions of love.

Often this bigotry and arrogance has had catastrophic results, Vioxx, Celebrex, Thalidomide, HRT to name a just few. Rest assured that, those tens of thousands of deaths were not caused by ND's, but MD's alone.

When you cite epidemiological data indicating that some therapy is causing more harm than good, you confirm that "western science" is indeed our arbiter of fact.

Why be science-based merely when it suits? Why not ditch the unproven vitalism and the anti-science marketing campaign in favor of a 100% commitment to the scientific method?

I'll just try to sum up some of the ND criticisms here. The ND's keep pointing out examples where MDs have made mistakes. THAT IS SCIENCE. We have data. We make decisions based on our best understanding of the data. Sometimes we are wrong and need to change our actions. It's bad to be wrong and yes, sometimes people die, but without constant evaluation and refinement of our knowledge people will just be dying of other things.

To all the ND med students and backers here, did you ever here that "ND's used to do X, but we no longer do because it was proven to not work?" How about "Homeopathic remedy X was improved over time based on research showing the original combinations could be improved?" Can you give examples? If not, do you seriously believe that NDs solved all the problems of illness a century ago and we've been going backwards since then? How does that square with the huge changes in lifespans and survival rates for many illnesses?

Andrea referenced dark matter which is something that describes the exact opposite of what you just said about Newtonian gravitational effects of things we can observe and understand.

And, as pointed out earlier, Andrea's understanding of dark matter was sorely deficient.

The hypothetical concept of "dark matter" came about because gravity wasn't acting entirely as it should based on our understand that you say "never fails." So we theorize something else is present, a "dark matter" that must exist to cause this abnormality and tie things back together in our heads logically.

So close to right, and yet so very very wrong. We hypothesized the existence of dark matter, and then we went out and found it. Catch up with the century in which you live. The confirmation of dark matter means that our understanding of gravity — Einsteinian, not Newtonian — was correct. The alternative to dark matter was modifying the gravity law, but because observations of the Bullet Cluster and elsewhere confirmed the presence of dark matter, we know that we don't have to cook up new equations for gravity. There's just more matter out there exerting the same old kind of gravitational force.

I think truth usually sits somewhere in between perceived realities.

Person A: "Death is never due to demonic possession."

Person B: "Death is due to demons possessing the body in roughly 50% of all cases."

In the spirit of compromise, C then declares, "Truth must sit between these two possibilities. Demons cause on average 25% of all deaths!"

Wow, and asthma attack remedy you have to take repeatedly, every 20+ min.
Odd, when I have an asthma attack, 20 sec is a VERY long time if I can't find my inhaler. 20 min is enough for Advair or similar to take care of it in one dose for 24 h.

@108 Andrea says, "You're forgetting that we are not only trained in "voodoo" but are trained in how to APPROPRIATELY diagnose and treat disease. If we see an emergency situation, we refer out.

What if it can't wait for a referral? What are you going to do for someone who passes out in a the waiting room in a hypoglycemic coma, or a child brought in with Koplick's spots?

By Tsu Dho Nimh (not verified) on 17 Apr 2009 #permalink

I don't question the importance of the scientific method. What I question is the fact that you believe that the Western Medical Model is aligned perfectly with the scientific model. I also question how data is collected before drugs are approved for human trial; evidently you feel that the thousands of deaths caused by drugs being released for human use, and their subsequent catastrophic results were odd exceptions. Actually, they are proof that the biomedical model often contradicts the data often at the expense of the patient.

Personally, I can't speak for other ND's, but I need to see the data and the biomedical pathway and risk before I prescribe anything. Personally I am skeptical of homeopathy, as were many of my classmates at Bastyr, and don't use it (Though I have patients who swear by it). Personally, I focus on diet and nutrition and the evolution of humans over time (to better understand how we evolved and what the best diet and supplementation would be). I also calm a lot of patients down who want to get off medicines that have been proven to be safe and effective and I tell them to stay on their meds!
In my own personal studies I look at the effects of the agricultural diets impact on human reproduction and birth weights vs a Hunter-Gatherer lifestyle that is several million years old. My research is on the effects of the modern diet on humans who have evolved for several million years on on a Hunter-Gatherer diet and lifestyle. Personally, I have never used a wet sock treatment, but I have learned about hydrotherapy and have seen some impressive results. Personally, I have to know the chemical constituents in a plant and why they, theoretically at least, work a certain way. Do they contain bioflavinoids, terpenoids, alkaloids? which ones? I always PARQ patients. I do a lot of research and read a lot of pub-med etc. I think this is fairly common for the average ND. I personally would not be the primary physician of the above patient. I personally have treated such patients with diet and lifestyle but only make recommendations to their cardiologist or internist and let THEM decide if they agree with the diet and exercise recommendations. They don't have time for this usually, and since I am not interested or capable of doing their job, they don't have a problem with me. I don't pull people from their meds, that is their doctors job s/he prescribed them, placed the stent etc. Again, this is how I work, but I don't speak for others ND's.

âIf it walks like a duck and quacks like a duck, I would call it a duck.â

If it talks like a bigot and hypocrites like a bigot, I would call it a bigot.

Bigotry isnât always about the ârace cardâ.

By Doug Cutler (not verified) on 17 Apr 2009 #permalink

PalMD are you trying to censor me? I try to post and you don't approve/post it here? Is this no longer an open forum?

By Matthew White (not verified) on 17 Apr 2009 #permalink

Easy, there, Matt. I'll look and see if you're stuck in the spam filter.

Wow...there was a ton of cool stuff in the spam filter, some of it a few days old. Coming out now.

Blake Stacey: "We hypothesized the existence of dark matter, and then we went out and found it. Catch up with the century in which you live."

Not correct, historically the turn of events were the opposite as what you said. You meant to say we observed the effects of dark matter on things we *can* and *have* observed. It *must* exist because if it didn't then we would have no explanation for our current understanding of mass and gravity.

Not to put too fine of a point on this but we *believe* dark matter exists and unfortunately we have *not* observed it in the 21st century. We have a telescope (the LSST) scheduled to come online in 2014 which one of the main goals is to hopefully *discover* dark matter but its only 2009 right now. (not calling you slow or anything).

Here, I took the path of least resistance and copied the first 2 sentences from wikipedia on dark matter as example for what dark matter is:

"In astronomy and cosmology, dark matter is hypothetical matter that is undetectable by its emitted radiation, but whose presence can be inferred from gravitational effects on visible matter. Dark matter is postulated to explain the flat rotation curves of spiral galaxies and other evidence of "missing mass" in the universe."

Pick whatever sources you want, they all say the same.

Mike, I don't know anything about cosmology, but Blake, I'm sure, will school us both.

Matthew, thanks for letting me know to release your comment. I enjoy its abrasive,um, what's the word..."wrongness". For example:

Since when do vitamins and fish oils and these other âtwigsâ commonly cause lactic acidosis like Biguanides?

Biguanides (metformin being the only available member of the family) have a long track record for safety, efficacy, and cost-effectiveness. Lactic acidosis is rare, and almost always do to prescribing it in the wrong patient (ie someone with a significantly decreases cratinine clearance). My impression is that you are hostile to it simply because it is in the standard medical toolbox instead of in one of your magic herb jars that you can sell to patients for much greater than the 4 dollars/month that metformin goes for, and without all that pesky safety and efficacy data.

Or Whitecoat Talesâ thorough knowledge of all things medicine (being a 3rd year MD student, wow!)

Yes you're right. I must not know anything. After all practicing voodoo for 20 years means you must know science!
Condescension does not become you.

when he states in comment 26 that âIf you want to be a pcp take boards and do a residency.â How about this: if you want to comment, do your homework first. (Thatâs a quote from me by the way). The truth is they do. Both of those things. The NABNE gives the board exams and you could find Naturopathic residencies if you tried. Iâll get this one out of the way for you. But their test isnât the USMLE so therefore itâs no good and shouldnât be considered as valid.

I am infact aware of the NABNE. They are not a valid test. Why?
http://www.nabne.org/nabne_page_23.php#Anchor5

Well to start with, from their own website, they test your knowledge of homeopathy. Magic potions containing no active chemical seems to conflict heavily with the pharmacology they're also supposedly being tested on. There is no further breakdown of the exam, just a list of subjects tested. Based on the licensed naturopaths commenting here, it's difficult to see how this exam could be comprehensive.

The existance of the exam is not sufficient proof that your exam prepares you to practice medicine. Infact, the licensed naturopaths were completely unable to answer basic questions that I (as you have so eloquently emphasized) a med 3, have to answer correctly every rotation.

The COMLEX isnât the USMLE either. Is it not valid? Have you, Whitecoat Tales, never seen a D.O.? (P.S. thatâs what the COMLEX is for, in case you didnât know).

Yes yes, we get it, Me big ND man, Me make young whitecoattales sound unreasonable.
DOs today practice evidence based medicine and follow guidelines similar to MDs. Indeed, COMLEX is extremely similar to the USMLE. How do I know? I studied for boards with my DO student friends, we compared notes.

Admittedly their osteopathic manipulation practices are often as not-evidence-based as anything a naturopath practices. However you'll find the majority of DOs don't spend much time of their practice on osteopathic manipulation. Of those that do - especially those who practice craniosacral manipulation - many are no different than chiropractors in their knowledge of real medicine.

Don't chiropractors take a licensing exam too? Would that make them medical doctors all of a sudden? Some chiropractors don't believe in the germ theory of disease, lets give them prescribing rights, and see how they treat pneumonia!

All that is going on here is that a small group of people are taking pot-shots. Like PalMD calling Caitlin naïve and regarding her as dangerous because she is âbucking those in your profession who have been at this longer.â

Caitlin said she's an MD student. MD students today are held to a pretty high standard on needing to learn evidence based medicine. She had better be familiar with the actual risks and benefits of those alternative practices, because just saying "they work" when asked about them by a patient is a hair shy of malpractice.

Whitecoat Tales, the glorious 3rd year student that he is, is bucking the same way at Dr. Morstein or Dr. Jen or any other licensed N.D. on this forum.

Argument from authority is an ugly thing. "Dr Morstein is a an older wiser naturopath and therefore the youngin must be wrong!"
We were having a discussion about whether the naturopaths are capable of practicing evidence based primary care. As it happens, Dr Morstein is less experienced at this than I am, because evidence based medicine is what I learn, not what she's learned.

On the other hand, you've complained that I'm throwing out pot shots, despite not having a single medicine based quibble about what I've said.

Now, put up or shut up, I posted on this forum an article by naturopaths on asthma care. Asthma is a disease with well established evidence based guidelines. Not following those guidelines can easily result in death. For an MD to break with those guidelines is malpractice.
Naturopaths want to be primary care doctors? Than get held to the same standards as everyone else. The article by Emily Kane published by the AANP disagrees with those guidelines on all points. You wanna tell me I know nothing because I'm a 3rd year med student. Go ahead, read the article, and either tell me "No this article is crap and not representative of naturopathy because of [reason]" or "yes this article is valid for [reason], furthermore it's better than/equivalent to beta agonists and inhaled corticosteroids because of [reason]"

You've been a huge blowhard in your comments attacking me, you've commented that I'm a 3rd year med student, and implied i'm not worth listening too multiple times. But not one of you naturopaths has addressed the specific medical questions we've brought up here.

I suggest this article by Dr Kimball Atwood on naturopathy in general - since students aren't prominent enough for you.
http://www.pubmedcentral.nih.gov/articlerender.fcgi?artid=1140750

Personally, I can't speak for other ND's, but I need to see the data and the biomedical pathway and risk before I prescribe anything.

All right -- applying this to "energy healing," "vital forces," etc. leaves you with -- hmmm.

It appears that you're basically a short-course PA with a lot of time wasted on stuff you don't use.

By D. C. Sessions (not verified) on 17 Apr 2009 #permalink

Yes I find it sad. So many people who seem to want to help patients, but so much time wasted on nonsense in an insulated, bizarro world.

On the other side of the fence we have doctors, nurses, physical therapists, occupational therapists, respiratory therapists, vocational therapists, radiology techs, EMTs, PAs, nurse practitioners, medics, psychologists, behavior therapists, mental health workers, case managers, pharmacists, and on and on and on, all joined by the common language of science and the practice of medicine. We get along. We work as a team. It's never dull.

Why would anyone go to a school where one is taught that vaccines are poison and doctors are assholes who claim to treat disease but only address the symptoms? Why seek to be alienated from the rest of the healthcare world?

I've had friends get sucked into cults like the Moonies. It seems like a similar phenomenon.

My apologies to Matthew White, in my response I implied he was an ND, rather than an OMS II. I was confused, because most OM students I know, know charlantry when it hits them in the face.

Thanks, Blake. Nice links, too.

By D. C. Sessions (not verified) on 17 Apr 2009 #permalink

On the issue of science vs Indigenous knowledge, I would argue that many cultures did use the scientific method in developing medicine and their tool kit. I would also argue that these could be compared with modern experimentation carried out based on hypothesis and modified. An example would be the Inuit Harpoon developed over time in the Arctic.http://pwnhc.learnnet.nt.ca/exhibits/nv/harpoon.htm The "theory" that bone could be used to fashion a tool for carrying out seal hunts led to the first early bone tools. Later "experiments" carried out in seal hunts led to refashioning and readjusting the original hypothesis until the modern tool emerged, which is fairly similar to the prehistoric one. The Inuit would not call themselves scientist. But, scientific method, using hypothesis, experimentation and modification of theory was carried out. The end result was a successful hunting technique that out competed other tribes in the area.
Whether or not this experimentation is the intellectual property Right of the tribe or community that developed it is the current reason why new laws are being developed as we speak regarding "biopiracy", and the use, by pharmaceutical companies to patten known community based medicine. the most famous case is actually taking place in India with the plant "Neem", which has been used for centuries as a fungicide. in a 1995 law suit against WR Grace. Similar attempts to gain legal control over medicines developed in the third world include, Hoodia, Tumeric and several anti-diabetic Herbs current used by local tribes in the Borneo rain forest. I refer you to the following pages. http://www.patentbaristas.com/archives/2005/12/13/india-fights-back-aga….
As regards the statement by white coat that he is concerned by the "Bloody present", state of Naturopathic medicine I am a bit confused, did I miss the bus? What blood where? ND's almost never get sued for Malpractice. As for the odd comments about energy medicine? and other New Age treatments. Sorry not at my med school. My first year was Anatomy with cadaver lab, biochemistry, physiology, histology, embryology, neurology one quarter of Chinese Medicine (which I don't practice), and ND philosophy. My second year was Pharmacy, pathology immunology, lab for blood draws, Physical exam skills and public health, homeopathy (not a fan personally)physical medicine, nutrition, botanical medicine, my third year was pediatrics, cardiology, gynecology, Gastro, EENT, emergency medicine internship, My fourth year was Urology, EENT, Internship liver and liver disease. Where are you getting your information about energy healing and vital forces??????
What vital forces? Are you talking about the bodies ability to self repair with proper nutrition and a good immune system? How is that a conflict with modern medical theory? Elderly people are less likely to survive flu season because their bodies ability to repair is often challanged. Since when did this become controversial?
Or are you talking about the early idea by scientist that mice could grow in an empty jar?

Dr Benway, we were not taught at my medical school that vaccines were poison??????????? HUH?
I took immunology with Heather Zwicky a Yale University professor.
What are you talking about??

On the other side of the fence we have doctors, nurses, physical therapists, occupational therapists, respiratory therapists, vocational therapists, radiology techs, EMTs, PAs, nurse practitioners, medics, psychologists, behavior therapists, mental health workers, case managers, pharmacists, and on and on and on, all joined by the common language of science and the practice of medicine. We get along. We work as a team.

It's not like there isn't plenty of work to go around, is it?

Frankly, I like working with people operating at higher levels than I do. Comforting, that. If I ever have a potential pericardial tamponade come in, I do not want to have to make that call (although at my level it's moot; one of us has a phone booth if he really wants to use it.)

On the other hand, I have no problem calling the shots on a field spinal immobilization with anyone. I know for certain that I've done lots more than the MDs I work with.

It's never dull.

I wouldn't quite say that -- as the saying goes, "hours of boredom punctuated by moments of screaming terror." Or, as I prefer to put it, "boring is good."

Why would anyone go to a school where one is taught that vaccines are poison and doctors are assholes who claim to treat disease but only address the symptoms? Why seek to be alienated from the rest of the healthcare world?

Well, being an MD doesn't immunize against assholiness. Fortunately, the ones I've known are competent assholes. I'll forgive a lot for that.

By D. C. Sessions (not verified) on 17 Apr 2009 #permalink

Whitecoat Tales:

Now, put up or shut up, I posted on this forum an article by naturopaths on asthma care. Asthma is a disease with well established evidence based guidelines. Not following those guidelines can easily result in death.

In case you didn't know, it did recently. A little girl ended up a little corpse thanks to a licensed practitioner of woo being unable to provide competent primary care (as in, not even oxygen) to a child with an acute attack of reactive airway disease.

There's a reason my canonical challenge to woosters is the cyanotic patient. I trust that all of the medically trained know their ABCs [1] -- and I've yet to meet any of the Woo Side who even know what I'm talking about.

[1] Including basic emergency care techs like me.

By D. C. Sessions (not verified) on 17 Apr 2009 #permalink

I find it funny that you are willing to pick and choose select passages to make fun of me on without being willing to post my entire comment in its entirety. Either post it so people can read it in its complete form or don't comment on it. No one has a good basis of what you're talking about unless you do. You run the fair risk of sounding like you are taking my comments out of context if you don't. Are you not willing to let others hear what I have to say? What are you afraid of?

By Matthew White (not verified) on 17 Apr 2009 #permalink

DocWimz,

Tell me what naturopathic program you attended. I'll go check out its website and get back to you about the vitalism.

Um, Matt, your comments ARE posted in their entirety. Where do you think we got the quotes from?

My apologies on this point. I couldn't find it earlier, I see it now.

By Matthew White (not verified) on 17 Apr 2009 #permalink

I apologize, but long comments and comments with multiple links get trapped in the spam filter until someone lets me know their comments are lost.

I find it funny that you are willing to pick and choose select passages to make fun of me on without being willing to post my entire comment in its entirety.

Holy succussion, you mean the the rambling, spittle-flecked screed that comprises comment #126 is an abridged version of the original??? Wow. Just...wow.

By Jennifer B. Ph… (not verified) on 17 Apr 2009 #permalink

@D.C.

I was not infact aware of that one. I've been trying to emphasize the easily avoidable acute situations in my woo-aimed comments because theres an easy obvious answer... that generally they won't get correct. However I didn't know of a documented confirmation of this.

@Matthew: How did you think I was getting quotes from your comment without your comment being posted?

@docwimz: As has been discussed elsewhere on this site, one of the reasons naturopaths aren't sued very often, is that there isn't a clear standard of care. There are very few position statements. In most states naturopaths aren't licensed to do much at all! There are also very few naturopaths. So malpractice has nothing to do with it.

To all the naturopaths: Do you guys get ACLS or PALS or ATLS certifications? If not, I don't see how you can be qualified to be 'doctors'
As D.C. Sessions has pointed out, those acute care protocols are exceedingly important.

Tsu Dho Nimh said:âWhat if it can't wait for a referral? What are you going to do for someone who passes out in a the waiting room in a hypoglycemic coma, or a child brought in with Koplick's spots?â

Disclaimer: I am not a doctor and I do not represent my school or my profession so donât take what I say seriously. I am only here for entertainment purposes. Yes, I admit this reflects I have no life. But who really has a life in medical school?

Scenario: Patient comes into office waiting for an appointment to address poor glycemic control that is clearly not being managed by current pediatrician. Patient is passed out when you come to greet him and his parents.

Immediate Emergency Action:

Have my MD assistant call 911

Assess ABCs and in secondary survey would look for diaphoresis, dilated pupils and pallor.

Dx: Severe hypoglycemia

Intravenous glucose bolus or injected glucagon (which mom should have been educated in having for patient at all times). This is also dependent on the âpracticingâ state. Should be looking for any indications of infiltration and any transient hyperinsulinaemic response.

Oxygen

Continue to check ABCs. Patient stabilizes. Make patient comfortable. Paramedics arrive and transport patient. Call ahead to local ER to give the status of incoming patient that will be safely monitored until cleared to go home. Chart notes like it was scripture and have a follow up plan reminder to look for neurological deficits. Follow up the next day with the ER doctors and parents. Have patient RTO for COMPLETE management of diabetes.

A PCP or any doctor is required to understand that not everyone in a coma is a diabetic. In this scenario, I would know the history of my new patient and would assume that this was more likely an iatrogenic cause. (Imagine that!).

Oh Noes! Is this a good time for a plug for Dr. Mona Morstein? If you are diabetic and want to keep your legs, eyes, nerves, kidneys and heart then go setup an appointment with a naturopathic expert in diabetic care.

Measles: Not a medical emergency and probably will never be seen in practice.

Only 29â116 cases were reported annually between 2000 and 2007. US Centers for Disease Control and Prevention. Measles â United States, January 1âApril 25, 2008. MMWR Morb Mortal Wkly Rep 2008;57:494-8.

Scenario: Patient PTC to office with cc: cough, fever, conjunctivitis. Temp 101. EENT exam reveals Koplikâs spots on buccal mucosa. Patient appears malnourished. Parents were advised from a âfriendâ to not vaccinate.

No worries, it is only in the prodromal phase and the maculopapular rash will come after the Koplikâs spots. But definitely advise to have no contact with other children, immunosuppressive individuals or pregnant women and keep much hydrated! Gatorade doesnât count.

Treatment: Vitamin A and etcâ¦â¦.(I will just name one, but NDs have a lot of effective modalities in their âwitch doctor bag of tricksâ).

Supplementation (Vit A, Vit E and zinc) resulted in an increase in total number of lymphocytes and measles IgG concentrations. Coutsoudis A. Pediatr Infect Dis J. 1992

A randomized, controlled trial of vitamin A in children with severe measles. Hussey GD, Klein M. N Engl J Med. 1990

Dx: IgM, IgG, or nasal swab

Measles sequellae are real, but very rare in the United States. Complete different story in underdeveloped countries. Why arenât the pharmaceutical companies making vaccines for these countries in mass dosages? Is it a profit issue, ha-ha like they canât afford the goodwill charity? How about we at least send loads of Vitamin A supplements and carrots? Whatâs up doc!

â90% of patients with severe measles had hyporetinemia. Supplementation reduced sequellae and increased weight gainâ. Coutsoudis A. Am J Clin Nutr. 1991

Orthodox medical treatment: NONE! Wait, what? You mean naturopathic physicians are and will continue to be legitimate and needed PCPs? Say it ainât so!

Well, I take that back, I am sure some medical âdoctorsâ will Rx. for antibiotics (viral right?) and aspirin. Reyeâs syndrome anyone?

Despite the blatant lies by Dr. Bengay (which I highly advise to skim past his bigoted posts), our school teaches the pros and cons of vaccinations. You see, we are not indoctrinated and we are trained to individualize our treatment of our patients. We do not have the pharmaceutical companies treating our patients by their distorted âguidelinesâ.

Ultimately, the decision to vaccinate is up to the parents with the education of the doctor. Personally, there are vaccines that absolutely need to be done, but done in a way that is safe. The parents need to be aware of the decisions to not vaccinate and the possibility of resulting diseases with pictures, stories and the end result of death (sometimes happening in one day). But vaccines like Hep B for infants? Are you freakin mad? End with a consent form that parents âreceived complete vaccination education and made their own choiceâ. Signature. Like it or not, there are modern day âwitchâ hunters so you have to protect yourself.

Now Tsu Dho Nimh, you made a great free offer and stated this: âMona Morstein wrote: I have not published this, which I probably should, I agree. Technical writing is not my forte, I admit. Repeating in case she misses the other post: Tell you what ... I'm an ex-medical technologist AND a good technical writer. I also live in Phoenix, AZ so there should be no problem with logistics.â

Have you made the bold initiative to call Dr. Morstein? Have you been calling around your local NDs to make your free proposal? I donât know, it kinda sounded like you were serious, but talk is very cheap! So get off your high horse and go do some good!

I know I am typing to a cement wall. But Naturopathic medicine is not on trial here. It has spoken for itself before orthodox medicine ever came along; it continues to speak for itself today and will continue to do so in the future. The âscientificâ world will catch up to it after they get their $#%$# together.

Hearing that echo still? âThe paradigm is changing my misguided friends.â

And btw if you feel I missed anything in the above scenarios please feel free to enlighten. I am very teachable.

By Doug Cutler (not verified) on 17 Apr 2009 #permalink

Not correct, historically the turn of events were the opposite as what you said. You meant to say we observed the effects of dark matter on things we *can* and *have* observed.

I meant to say exactly what I said. Telling me otherwise. . . with that kind of arrogance, you could go to med school.

Yes, dark matter is only observable indirectly. You could say the same thing about ultraviolet paint or the inside of a brick.

our school teaches the pros and cons of vaccinations.

Doug I know you went to my blog, you commented there! Did you read? I have a series of posts going about Vaccines, including their safety profile, with references. So now you give me a "balanced" breakdown of the "pros and cons of vaccines" and explain when they AREN'T appropriate.

You see, we are not indoctrinated and we are trained to individualize our treatment of our patients.

Vaccination is a public health issue, not just an individual issue. Talk to me about herd immunity.

We do not have the pharmaceutical companies treating our patients by their distorted âguidelinesâ.

There's that pharma shill gambit again. It doesn't work now, it didn't work then, it will never work.

But Naturopathic medicine is not on trial here. It has spoken for itself before orthodox medicine ever came along

Religous beliefs aside, evidence is important. Naturopathic medicine doesn't do evidence well. If you feel naturopathic medicine does evidence well, respond to my post concerning Emily Kane's AANP published article on asthma treatments.

But Naturopathic medicine is not on trial here.

If it ain't now, it will be soon. Words like "malpractice" and "titanic fucking incompetence" dance in my head like sugar plums.

It has spoken for itself before orthodox medicine ever came along

In a plaintive, ineffectual whine.

Measles sequellae are real, but very rare in the United States. Complete different story in underdeveloped countries. Why arenât the pharmaceutical companies making vaccines for these countries in mass dosages?

I'm not a fan of the pharmaceutical companies, but this is one area where you are incorrect. The issue is not lack of vaccines being sent to africa, but political issues - the vaccines not getting from point of entry to point of distribution, and distribution issues since you must maintain the cold chain for these vaccines.

Measles: Not a medical emergency and probably will never be seen in practice.

Theres that fallacy of induction agian. Naturopaths struggle with that one.

Unfortunately, do to the antivaccine crowd, another popular target on this blog, it's entirely possible we will see a resurgence of measles in this country.

Matthew White,

I think of DOs and MDs as equally competent, as both train in the same hospital based residency programs.

I don't think of NDs as equivalent to either, as their profession rejected the standards set by the Flexner report concerning science. For the last hundred years, they've viewed themselves as theologically opposed to modern medicine. And they haven't been shy in telling patients not to trust MDs.

NDs don't seem speak the same language as MDs or DOs. When they talk about "prevention," for example, they typically don't mean vaccines.

A number of NDs seem intent on getting the public to believe they're just like MDs or DOs. When NDs say they go to "medical school" and "residency," the public will assume they mean the same medical schools and residencies as MDs and DOs.

Oh, forgot to say: the "gotcha" in the land of the gotcha isn't PalMD. He's a pussycat.

No, the real scary gotcha is the thing you do that causes harm to a patient.

And by "you" I don't mean you personally.

I find it funny that you are willing to pick and choose select passages to make fun of me on without being willing to post my entire comment in its entirety.

It's upthread. This is the Dao of online discussion, and you may as well get used to it because it ain't gonna change just because you cry.

Either post it so people can read it in its complete form or don't comment on it.

Hate to break the news to you, Bucky, but you ain't the moderator here and you ain't makin' the rules. Learn the local customs or we may mock you again.

By D. C. Sessions (not verified) on 17 Apr 2009 #permalink

Blake Stacey, I am an ND, and I think "the Secret", is a load of crap, not to mention sick and abusive.

Oh, I promised myself that I was going to quit commenting on the insanity of this blog...but I just can't resist...

D.C. Sessions: I was curious about the licensed Naturopathic Doctor who did not offer appropriate care for the child you referenced above. Since you gave no link about this unfortunate incidence, I did a quick search on Google for "Naturopath and asthma related death". Strangely enough, the first article that popped up was an article by Cornell University, "Common asthma inhalers cause up to 80 percent of asthma-related deaths, Cornell and Stanford researchers assert." http://www.news.cornell.edu/stories/June06/AsthmaDeaths.kr.html

Now, I'm not going to go into how iatrogenic causes of death are one of the leading causes of death in the US. That would just be too easy. But, don't pick one instance where a doctor didn't do his/her job and forget the thousands of lives that have been lost in the hands of MDs. Who was it that was talking about bloody pasts and bloody present? Well, looks to me like you guys still have quite a lot of blood on your hands.

By the way, since I STILL have not found a reference to the licensed Naturopathic Doctor and the asthma-related death, would you please share with us a link?

By Andrea LaCoss (not verified) on 17 Apr 2009 #permalink

Matt's comment got caught in the spam filter, and wasn't released until he expressed his feeling that i might be censoring him. When it was released, it appeared in its original position upthread.

I rarely censor comments---I have had one or two commenters over the years who i've had to kick to the curb for hijacking threads beyond any useful discussion, for making hate-filled or dangerous comments, or for trying to explicitly sell things. The ND folks commenting are staying engaged and (with the possible exception of doug, who appears to be some sort of drooling half-wit) are helping to clarify this issue for me.

Oh, I suppose that since I mentioned iatrogenic causes of death being one of the leading causes of the death in the US, I should provide a reference for that...so here you go! Enjoy!

http://www.alternative-doctor.com/specials/Dr_%20Barbara_Starfield.htm

Now, I know that it looks like it's from an "alternative doctor", but Barabara Starfield is from Johns Hopkins. If you want the original you can go find it in JAMA.

By Andrea LaCoss (not verified) on 17 Apr 2009 #permalink

PalMD: Glad to hear you are finally realizing there is a difference between "naturopaths" and naturopathic physicians.

Keep up with the speed there big guy. :)

Censor me if you must, I will understand.

By Doug Cutler (not verified) on 17 Apr 2009 #permalink

Pope's "Essay on Criticism" seems appropriate, notably verse 16:

Assess ABCs and in secondary survey would look for diaphoresis, dilated pupils and pallor.

Dx: Severe hypoglycemia

I am beyond impressed that you might be able to diagnose insulin shock (as distinct from, e.g. diabetic ketoacidosis) based on this alone. I'll leave those who can diagnose to list some of the alternate causes with the same presentation you've given.

Intravenous glucose bolus or injected glucagon (which mom should have been educated in having for patient at all times). This is also dependent on the âpracticingâ state. Should be looking for any indications of infiltration and any transient hyperinsulinaemic response.

IV glucose? Boggle. I note several very basic things that you're not checking, and that you're depending on family to tote around IV glucose (which is not part of the family kit) even though you don't have it yourself at your PCP office.

Condemned from your own words.

Oxygen

And this is going to do -- what? -- to the physiology of a patient in insulin shock? How about a patient in diabetic coma? Do you expect the results to be the same?

Continue to check ABCs. Patient stabilizes.

What if the patient doesn't? Come on now -- you're the primary care provider. You need to have an answer for that question or you're just a poser reciting a script like any telemarketer.

Make patient comfortable. Paramedics arrive and transport patient. Call ahead to local ER to give the status of incoming patient that will be safely monitored until cleared to go home. Chart notes like it was scripture and have a follow up plan reminder to look for neurological deficits. Follow up the next day with the ER doctors and parents. Have patient RTO for COMPLETE management of diabetes.

Except that, as noted above, the patient may not have followed your script. Even if she did, you didn't do several things that are SOP for proper prehospital care and can make a major difference to follow-up by the patient's endocrinologist.

Come on, chief -- I'm just a nonurban first responder and I get refreshed on this one every year. It's utterly basic BTLS stuff -- and you flunked.

A PCP or any doctor is required to understand that not everyone in a coma is a diabetic. In this scenario, I would know the history of my new patient and would assume that this was more likely an iatrogenic cause. (Imagine that!).

And by making that assumption you may well have killed your patient because you were just following a script and jumped ahead from partial information to a foregone conclusion.

Not every unresponsive diabetic patient is unresponsive due to the diabetes. You blithering nitwit, you didn't even check her blood pressure or O2 saturation before you charged off on IV glucose (!!!!).

You're claiming a higher level of care than freaking BTLS, and you can't even do the basics of that correctly.

By D. C. Sessions (not verified) on 17 Apr 2009 #permalink

Oh I get it, PalMD, you only find research to be noteworthy when it works out in your favor. What you sent me were blog posts, with no real references to debunk the assertion that iatrogenic causes of death are the 3rd leading cause of death in the US. You expect me to believe a blog post over JAMA? I thought you guys were all "Hail JAMA!"?

So, primitive or not, numbers don't lie. You can explain them all you want, but numbers don't lie.

My point in my earlier post was not that we should do away with Conventional medicine based upon the stats. I, for one, support integrated medicine, not segregated medicine. Furthermore, I understand that there are many many people who are helped and whose lives are saved by MDs. My point was that people on this blog shouldn't point fingers when they have their own skeletons in the closet (and many of them).

By Andrea LaCoss (not verified) on 17 Apr 2009 #permalink

Strangely enough, the first article that popped up was an article by Cornell University, "Common asthma inhalers cause up to 80 percent of asthma-related deaths, Cornell and Stanford researchers assert." http://www.news.cornell.edu/stories/June06/AsthmaDeaths.kr.html

Now, I'm not going to go into how iatrogenic causes of death are one of the leading causes of death in the US.

I do so love this line of reasoning: patient abuse of a medication that is so effective that people don't die of the disease but instead die of misusing the medicine becomes proof of how bad the medicine is.

It's rather obvious that you've never had to care for anyone with a severe case of asthma. While you're attending the University of Google, you might look up the side effects and contraindications for inhaled albuterol sulphate. Also look up the OTC alternative that people use when their prescription inhaler runs out and its side effects and contraindications.

When you do, get back to us with the issues and alternatives faced by an asthma patient who has a viral respiratory infection during peak allergy season.

Meanwhile you can look up the case of Megan Wilson and her licensed "ND," Lucinda Messer.

By D. C. Sessions (not verified) on 17 Apr 2009 #permalink

Tsu Dho Nimh said:âWhat if it can't wait for a referral? What are you going to do for someone who passes out in a the waiting room in a hypoglycemic coma, or a child brought in with Koplick's spots?â

Scenario: Patient comes into office waiting for an appointment to address poor glycemic control that is clearly not being managed by current pediatrician. Patient is passed out when you come to greet him and his parents.

No, I didn't say you knew the patient had poorly controlled diabetes. I said the patient is passed out in the waiting room ... I know it's hypoglycemia, but all you know is that your patient is lying there.

Intravenous glucose bolus or injected glucagon (which mom should have been educated in having for patient at all times).

MOM? You are going to depend on MOM to have the right stuff? You are planning to be a primary care practicioner and you don't even have a glucose meter?

And BTW, standard ER protocol is to quickly draw some blood for chem tests or get a fingerstick at least, then shoot up the glucose ... if that doesn't work by that time the quick glucose test is back and you can estimate insulin needs. Elapsed time should be under 2 minutes.

(I assume your assistant or you can run a quick glucose test, and slap on an oxymeter)

Measles: Not a medical emergency and probably will never be seen in practice.

Au contraire, Doug. You are in Arizona which had an outbreak last year brought in by a Swiss tourist, and your clientele will be heavily drawn from the "what, me worry" crowd who confuse veggies for vaccines.

No worries, it is only in the prodromal phase and the maculopapular rash will come after the Koplikâs spots. But definitely advise to have no contact with other children, immunosuppressive individuals or pregnant women and keep much hydrated! Gatorade doesnât count.

No worries, oh no, none at all. This is a disease that puts 20-30% of the victims (regardless of nutritional state) in the hospital for treating the pneumonia or diarrhea or encephalitis. And it's contagious as hell ... to the extent that any child entering your office for the next hour, like maybe the cute one with asthma who can't afford another round of pneumonia, is very likely to be infected.

It's a reportable disease, BTW and the health department can make your life really miserable if you fail to report promptly. You forgot to notify the health department, preferably by phone from your office BEFORE the child leaves.

Why arenât the pharmaceutical companies making vaccines for these countries in mass dosages? Is it a profit issue, ha-ha like they canât afford the goodwill charity?

They are trying to vaccinate in those countries, you blithering nitwit. Measles has a problem in that it's a live virus, has to be kept cold or it goes bad.

Orthodox medical treatment: NONE!

Yes, there is ... in areas where Vitamin A levels are known to be low (Africa, India, etc.) a hefty shot of that. Standard supportive care (IVs for the dehydration, O2 for pneumonia, antibiotics for the secondary bacterial infections) is what got the mortality rates down before the vaccine.

Wait, what? You mean naturopathic physicians are and will continue to be legitimate and needed PCPs? Say it ainât so!

It ain't so. I'd rather trust a veterinarian.

Well, I take that back, I am sure some medical âdoctorsâ will Rx. for antibiotics (viral right?) and aspirin. Reyeâs syndrome anyone?

What rock have you been living under? Aspirin for children with fever has been contraindicated since 1980, when the CDC started warning physicians and parents about the connection with Reyes Syndrome.

And antibiotics are prescribed if the complications include a secondary bacterial infection, which is very likely because measles virus suppresses the kid's immune response.

vaccines like Hep B for infants? Are you freakin mad?

No, I'm freaking sane. An infection early in life is extremely likely to be come chronic and lead to cirrhosis and liver cancer and an early death, as well as being a source of infection for others. Body fluids include spit, snot and poop and those are spread liberally by all toddlers, along with a little blood at times.

Have you made the bold initiative to call Dr. Morstein? Have you been calling around your local NDs to make your free proposal? I donât know, it kinda sounded like you were serious, but talk is very cheap! So get off your high horse and go do some good!

I was doing good today. I was writing a bid for translating 300,000 words of medical-ese into English for a clinic. If I haven't heard from Tiffany or Mona by the end of next week, I'll ask again.

By Tsu Dho Nimh (not verified) on 17 Apr 2009 #permalink

I'm in ur spam filters.

Before we round up the criminals breaking rules and killing patients on the MD side and the ND side, we first must establish what those rules actually are.

Here's one from the MD side: "Thou shalt not overstate thy data."

Is that a rule on the ND side also?

Here's one from the MD side: "Thou shalt not overstate thy data."

Is that a rule on the ND side also?

You're a sweet little titmouse, and all in all I'm growing quite fond of you.

However, that was some damn fine Ardbeg you just ruined, and if this weren't a spray-resistant ThinkPad I'd be out a laptop. Affection will only get you so much forgiveness.

By D. C. Sessions (not verified) on 17 Apr 2009 #permalink

Since this little chit chat started with a theoretical case, I'd like to post a real case and ask for treatment plans.
I'm a naturopathic student and today at one of our free clinics my patient was a 7 y/o boy whose chief complaint was a disabling phobia of wind. He has almost daily panic attacks when wind brushes his clothes or body and even when he just hears or see wind (this might be trees blowing or wind sounds through an open window). The phobia is so disabling that he frightens other children at school and is spending hours locked in a bathroom where there are no windows to see or hear weather. A paternal grandmother and paternal cousin who he has never met also suffer from phobias but not as pronounced.
Parents have taken the child to counseling & allopathic care. Mom reports son's phobia was not relieved by SSRIs (several tried by psychiatrist) but had some numbing of panic symptoms on benzodiazepines. Even on meds he was unable to go outside on any day that was breezy and had numerous school absences. Currently, he is taking no meds because the benzos made him extremely fatigued & sleepy and he was unable to complete coursework or listen to the teacher.
Physical exams were normal. Patient was compliant and easy going but very much embarrassed by his phobia and frequent panic. He became greatly distressed when the visit was finishing up and he began to anticipate having to go outside on a fairly windy day.
This being a naturopathic clinic we prescribed Aconite 6c QD and PRN. He is following up in 2 weeks. I have no experience treating such conditions but my attending doc has had truly miraculous results in a similar time frame. I will post an honest follow-up when I get one, even if there is no improvement or regression.
In this case which treatments would people (NDs, MDs, whomever) have chosen?

SS wrote: "this being a naturopathic clinic we prescribed aconite 6c QD and PRN."

That's homeopathy.

Aconite is poisonous in its actually present form.

I thought you said you treated the child.

-r.c.

Aconite is a placebo treatment.
SS in your classes have you learned pharmacology? Please try to reconcile dose response curves with homeopathy.

Blake Stacey: "Yes, dark matter is only observable indirectly. You could say the same thing about ultraviolet paint or the inside of a brick."

But you do realize the gaping differences between breaking open a brick to see whats inside vs always hypothesizing and theorizing about it correct? We have probability and tangibility with UV paint so that is just another bad comparison. There is extremely important differences between discovery, hypothesis and theory that should not be blurred on faith. In the case of dark matter what happens if the LSST comes online, we discover dark matter and it doesn't account for all the deviation we've been calculating? Do you consider that the very point in time where we discovered "darker matter?" That makes no sense scientifically even though I do understand it is a common mistake for overly confident "scientists."

When you break open a brick, all you see is a new outside.

When you break open an Internet argument, all you get is bile and choler. (Oh, come on, somebody has to speak truth to power and stand up for the humeral-balance theory of medicine. It's been going strong for two thousand years. Someday, those nasty allopaths will see the light. And I'll bet they'll be pretty melancholy when they do.)

In the case of dark matter what happens if the LSST comes online, we discover dark matter and it doesn't account for all the deviation we've been calculating?

Then the astrophysicists have job security for a while longer.

Do you consider that the very point in time where we discovered "darker matter?"

I don't know how you managed to construe that from what I've written so far.

Also, your names are lacking in the imagination department. I'd go with "matter the darkness of celestial obsidian".

Daijiyobu and Whitecoat Tales: How about instead of mocking what SS and her attending did for the patient and actually, oh I don't know, ANSWER THE QUESTION? Oh, wait, is that because how you would have treated it is with an SSRI or Benzo? Hmm...bummer...what ever do you do when the drugs don't work?

By Andrea LaCoss (not verified) on 17 Apr 2009 #permalink

D.C. Read carefully. No, I am mean read REALLY carefully. But the reality is that no matter how hard you read, you already made the conclusion about me because I am a naturopathic medical student.

âI am beyond impressed that you might be able to diagnose insulin shock (as distinct from, e.g. diabetic ketoacidosis) based on this alone. I'll leave those who can diagnose to list some of the alternate causes with the same presentation you've given.â

Tsu Dho Nimh wanted to know what would be done in a situation with a hypoglycemic coma in an office setting and not in a hospital with acute care. I knew my patientâs history already with the parents there with me so I made it a diabetic scenario â hence my reference to Dr. Morstein. Did you miss that? Diabetic ketoacidosis can become a hyperglycemic coma and this was a hypoglycemic coma scenario. Still following?

âIV glucose? Boggle. I note several very basic things that you're not checking, and that you're depending on family to tote around IV glucose (which is not part of the family kit) even though you don't have it yourself at your PCP office.â

Well please note the several things that I missed not checking. Yes IV glucose. NDs use IV therapies and an IV is at our disposal. Yes, an injected glucagon emergency kit that can be used in the office or by a parent. They even have them in day cares these days, but I will make sure to talk to the manufactures of the âfamily kitsâ to get one in. In the meantime what does that have to do with this patient? And again I was making up a scenario so wow you are correct, I donât have one at my PCP office yet. I am still in school compadre.

âCondemned from your own words.â

Haha okay that at least made me laugh.

âAnd this is going to do -- what? -- to the physiology of a patient in insulin shock? How about a patient in diabetic coma? Do you expect the results to be the same?â

The oxygen wonât bring him out of his coma, the glucose will. If you have oxygen on hand (which I will in my future office) then you use it. Do we really need to discuss the benefits of oxygen on the body? Why wouldnât you use oxygen?

âWhat if the patient doesn't? Come on now -- you're the primary care provider. You need to have an answer for that question or you're just a poser reciting a script like any telemarketer.â

Oh, so you are adding to my scenario now I see. If the patient doesnât stabilize (come out of coma) then I continue to check ABCs until the paramedics arrive. And how nice, he has an IV already in place and is on oxygen.

âExcept that, as noted above, the patient may not have followed your script. Even if she did, you didn't do several things that are SOP for proper prehospital care and can make a major difference to follow-up by the patient's endocrinologist.â

My patients arenât going to follow that script? Thank you Captain Obvious! You are definitely amusing and it was a he and not a she, but thanks for following along carefully. And please do list the âseveralâ things because as I said, I was teachable, but so far you havenât taught me one thing. Well, other than family kits donâtâ have glucagon in them.

âCome on, chief -- I'm just a nonurban first responder and I get refreshed on this one every year. It's utterly basic BTLS stuff -- and you flunked.â

The publicâs emergency health care is in your hands? Scary.

âAnd by making that assumption you may well have killed your patient because you were just following a script and jumped ahead from partial information to a foregone conclusion.â

Again for the 14th time. From knowing the patientâs history and parents, I made the diagnosis. And yes I know it will be a shocker to hear, but the #1 cause is iatrogenic.

âNot every unresponsive diabetic patient is unresponsive due to the diabetes. You blithering nitwit, you didn't even check her blood pressure or O2 saturation before you charged off on IV glucose (!!!!).â

Okay again, thank you Captain Obvious, if you were following along my reasoning and that I knew the patientâs history then I would know it was from his diabetes. The âtwitâ name was already used on me by PalMD so come up with originality.

I wouldnât know the full status of patientâs liver function so please explain the BP and O2 saturation as he has no minutes to spare and needs glucose stat to avoid brain damage or death. I will listen to this very carefully.

âYou're claiming a higher level of care than freaking BTLS, and you can't even do the basics of that correctly.â

Show me where I made that claim.

Whitecoat Tales:

Sorry for skipping your questions, too many are piling up, but I will check out your articles and give you answers soon enough.

Do me a favor in the meantime and give an assessment of the above scenario because I honestly want to make sure I get it 100% right. I can see you would be the only one to be fair and honest on this entire blog. And really I donât think anyone else would know.

By Doug Cutler (not verified) on 17 Apr 2009 #permalink

I also find it remarkably odd that anyone would seriously offer "tangibility" as a criterion for judging that which never detectably impinges upon the daily lives of human beings. It's an entirely subjective gut reaction, founded on nothing but personal bias. By what standard are, say, radio waves "tangible"? Without instrumentation of some kind, I find they're a good deal less "tangible" than the hallucinations I get when I haven't slept in a couple nights.

And "probability"? Please.

Your words serve to prop up your comforting narrative that physicists are arrogant obsessives indulging in pseudoscience, but they don't really add up to an argument.

You know, maybe I'm going about this all wrong. I should have started off in the same spirit as PalMD's original challenge, way up at the top. Hmmm. Let's see.

Patient is a 19-year-old male presenting with a Dirichlet brane of codimension p. How would you characterize his spectrum of massless excitations? Is the patient tractable via light-cone quantization?

Very creative writing Blake Stacy. Sure you're in the right career path? Blogging like that has gold stars stamped all over it.

I'll explain breaking a brick open with two words common to the scientific community. (which I'm not even involved but somehow I seem to understand and you don't) Its called "probability" and "repeatability" regardless of your semantic gymnastics. Now how are you going to break open a "dark matter" when you haven't even discovered it yet?

"Probability".

You keep using that word. I do not think it means what you think it means.

You know who uses probability? And statistics? The astronomers who map the distribution of dark matter.

But why are we sitting here arguing about it? You've got a patient with a possibly unstable brane on your hands!

SS, you may want to refer your patient to a therapist that specializes in in Phobias. I am impressed by the "desensitization" technique; whereby the therapist slowly exposes the patient to the phobia until they gain confidence. SSRI's in children have been linked to side effects including, violent mood disorders and suicide, and many MD's are shying away from prescribing them to the young for good reason. I am not, personally, a fan of Homeopathy. But, if the child believes that he/she can take a pill to treat the phobia they may benefit and there isn't a risk of side effects that we would see with the SSRI's. Since the phobia is in the child's mind ie is an imagined fear. One could argue that placebo could benefit such a patient. I think the phenomenon of placebo affect has been studied and has value as a therapeutic tool. It might with your patient and should not be dismissed. That sugar pill might just help.

Tsu Dho Nimh: âNo, I didn't say you knew the patient had poorly controlled diabetes. I said the patient is passed out in the waiting room ... I know it's hypoglycemia, but all you know is that your patient is lying there.â

I came up with a scenario. You came up with the conditions. Be creative Tsu, thatâs what " good writersâ are supposed to do right? And it also helps for âwritersâ to use spell-check. Dr. Morstein wouldnât be impressed with mizspelzings.

âMOM? You are going to depend on MOM to have the right stuff? You are planning to be a primary care practitioner and you don't even have a glucose meter?â

Because you know, âMOMâ will be carrying me in her pocket everywhere she goes. It is called âdocereâ where the doctor teaches the patients. Is that a lost art theses day? And the parents need to know what to do no? I will get a glucose meter, stick to the scenario.

âAnd BTW, standard ER protocol is to quickly draw some blood for chem tests or get a fingerstick at least, then shoot up the glucose ... if that doesn't work by that time the quick glucose test is back and you can estimate insulin needs. Elapsed time should be under 2 minutes.â

You being serious? Run blood chem tests and send it off to the lab? I already know he needs glucose. Yes, a blood glucose meter would be needed in this scenario. I missed that.

âAu contraire, Doug. You are in Arizona which had an outbreak last year brought in by a Swiss tourist, and your clientele will be heavily drawn from the "what, me worry" crowd who confuse veggies for vaccines.â

So you are claiming every PCP saw this outbreak? Sources.

âNo worries, oh no, none at all. This is a disease that puts 20-30% of the victims (regardless of nutritional state) in the hospital for treating the pneumonia or diarrhea or encephalitis. And it's contagious as hell ... to the extent that any child entering your office for the next hour, like maybe the cute one with asthma who can't afford another round of pneumonia, is very likely to be infected.â

No I wouldnât be worried. There are 3 stages to the disease. Koplikâs spots are in the second stage. I also know my treatments would work. Read the Vitamin A studies again with the sequellae. NDs have used the same treatment with success. There are no guessing games if your treatment is going to work. You have to know it will!

âIt's a reportable disease, BTW and the health department can make your life really miserable if you fail to report promptly. You forgot to notify the health department, preferably by phone from your office BEFORE the child leaves.â

Fair enough that would definitely be required. I will remember that thanks.

âThey are trying to vaccinate in those countries, you blithering nitwit. Measles has a problem in that it's a live virus, has to be kept cold or it goes bad. â

Show me their âgoodwillâ. Sources! Since you know I am âpharm shillâ. Which I am definitely warming up to the name so I will keep it. The "nitwit" has already been used 3 times on me so need to follow your fellow cronies. Be original since you say you are a âgood writerâ.

âYes, there is ... in areas where Vitamin A levels are known to be low (Africa, India, etc.) a hefty shot of that. Standard supportive care (IVs for the dehydration, O2 for pneumonia, antibiotics for the secondary bacterial infections) is what got the mortality rates down before the vaccine. â

Sources for Vit A being used CURRENTLY in Africa, India. Standard supportive care isnât treatment, but nice try.

âIt ain't so. I'd rather trust a veterinarian.â

So he/she can keep you on your high horse!

âWhat rock have you been living under? Aspirin for children with fever has been contraindicated since 1980, when the CDC started warning physicians and parents about the connection with Reyes Syndrome.â

Nice google quote.

âAnd antibiotics are prescribed if the complications include a secondary bacterial infection, which is very likely because measles virus suppresses the kid's immune response.â

No kidding? We are talking about measles treatment. Antibiotics are not used to treat measles, but it sounds like you would be one of those docs that would be prescribing antibiotics and aspirin for measles.

âNo, I'm freaking sane. An infection early in life is extremely likely to be come chronic and lead to cirrhosis and liver cancer and an early death, as well as being a source of infection for others. Body fluids include spit, snot and poop and those are spread liberally by all toddlers, along with a little blood at times.â

Since toddlers are drug users and have sexual partners too.

âI was doing good today. I was writing a bid for translating 300,000 words of medical-ese into English for a clinic. If I haven't heard from Tiffany or Mona by the end of next week, I'll ask again.â

Well great, I am glad to hear. I hope you are being honest, but I guess I will hear about the calls that were made soon enough. Didn't know "professional" proposals could be done over the phone though. And just a friendly suggestion: Ex-medical technologist shouldnât pretend to be a doctor.

By Doug Cutler (not verified) on 17 Apr 2009 #permalink

Is beginning to think that "the white coat underground" is truly an underground and is saturated with wannabe doctors.

Whitecoat Tails is the only exception. He will make an excellent doctor.

By Doug Cutler (not verified) on 17 Apr 2009 #permalink

Don't worry Blake Stacey, nobody would trust me with an unstable "brane" and I'm happy to be absolved of that responsibility. I'm a mere code monkey. But you say that I think physicists are "obsessives indulging in pseudoscience" when my own sister is in her PhD candidacy for physics right now. Do I hate astronomers? My sister has dual major undergrad with that too as well and my brother works under the shuttle program. Maybe you think I hate MD's? My grandfather is a MD and my dad is a DMD.

Not the best reading of me Blake. I like everybody just fine up to and including Naturopathic Doctors that graduated from DOE accredited Naturopathic Medical schools. Maybe because I know the difference between them and the un-NDs. Don't get me wrong though, I don't always agree with them either just like I don't always agree with conventional medicine. I think there are pros and cons to either approach based on the situations at hand.

Docwimz wrote [#193]: "[I am an ND] I am not, personally, a fan of homeopathy [...] personally I am skeptical of homeopathy [...] and don't use it."

Why?

Do you NOT think that homeopathy is the "clinical science" that NPLEX Part II labels it (see http://www.nabne.org/nabne_page_23.php )?:

"the Core Clinical Science Examination is a case-based examination that covers the following topics [...including] homeopathy."

If yes "science", why? If "not science", why? You seem to be at odds with something your 'profession' expressly labels "science". Correct me if I'm wrong, but are you admitting that homeopathy is inert? So, how is that "clinical science"? Doesn't that destroy the integrity of the knowledge-type label "science"?

And per: "that sugar pill might just help", is it ethical to deceive a patient?

Oh, and per your:

"where are you getting your information about energy healing and vital forces? What vital forces? [...] tell me what naturopathic program you attended. I'll go check out its website and get back to you about the vitalism."

Are you THAT lost? Really, now: I refer you to the North American ND consortia statement (see http://www.fnpla.org/philosophy.htm ). It's not about any particular ND program, it applies to ALL of them. Are you sure you understand your creed, in its essential state?

The Textbook of Natural Medicine, 3rd ed. states the "life force" to be -- and ISYN -- a "physiologic system". That chapter by Zeff, Snider, & Myers is a free pdf download at http://www.naturalmedtext.com/ , for anyone interested. Specifically, here's the mandatory vitalism of where I went, claimed as fact (see http://web.archive.org/web/20041224053526/http://www.bridgeport.edu/ub/… ).

-r.c.

@PalMD: Your âchallengeâ has miserably failed. Your internist view of the âwholeâ body is like Wonder White Bread ©. Which you would believe would be âgoodâ for the body because of the synthetic and processed âenhancementâ.

The next challenge for you is to take a vacation from your blogging bigotry deception, aerobics and some âtwigâ tea to deflate that omniscient ego.

Are your employers (The Science Network) âpharma shillsâ too? The Science Network who you âwriteâ for shows that you are the true hypocritical charlatan. Damning and actually, very good videos about The Science Network which you have avoided like the plague. Watch all videos for complete context @ http://video.google.com/videoplay?docid=-7758662442132419447&hl=en

If it talks like a charlatan, and deceives like a charlatan. I would call it a charlatan. But your secret gadianton works will be revealed and your transgressions will be paid for.

Ironically, a family PCP and an internist almost took away my life and I would be in an orthodox medical school today if a naturopathic physician didn't restore my health. Your blatant, distorted and cavalier view of medicine will do the same with your patients. I look forward to seeing you on the other side.

@Bengay: Your posts belong on 4chan.org (the cancer of the internet) where young teenagers have thwarted the English language . There, they with decaying minds will at least understand your /b/ âlingoâ.

@Wannabe doctors. Wannabe scientists. Wannabe health professionals. Wannabe superstars. This was for entertainment purposes, but you made it boring. Have fun playing doctor.

@WhiteCoat Tails: Your âmy motivationâ blog said it all on what a doctor should truly be. I am going to hold you to what you have said through your blog and through the âstatementsâ you have said in private email because your patients will require and will deserve the same.

I will also continue to hold correspondence through email to avoid the unnecessary clutter that does not benefit our patients. I hope and pray you donât follow this bigot road as your mentors, donât disappoint my âolive branch of trustâ in you.

@Mankind: "Let us not waste our time in idle discourse! (Pause. Vehemently.) Let us do something, while we have the chance! It is not every day that we are needed. But at this place, at this moment of time, all mankind is us, whether we like it or not. Let us make the most of it, before it is too late!" - Samuel Beckett, Waiting for Godot

I am hearing that echo more loudly than ever now! âThe paradigm is changing my misguided friends.â

dcutler@scnm.edu for anyone else with the desires to put patients first.

By Doug Cutler (not verified) on 18 Apr 2009 #permalink

Tales
Whitecoat Tales
As in stories. I realise it's not that clever a pun, but it's the only one I've got.

Doug: I'm glad you enjoyed my post, and our correspondance, but I hope you aren't misinterpreting my politeness in those forums as agreement on naturopathy. It's my acknowledgement that naturopaths have good intentions. Part of my post explains my motivation for demanding evidence, precisely because good intentions aren't enough. And evidence is exactly what naturopathy has not shown me.

I wish you guys would quit slinging the word bigot around. It's disappointing, because instead of criticizing "the western model" and just trying to prove your point, you've switched to ad hominem.

@docwims: Don't you see an ethical dilemma with prescribing sugar pills? How do you reconcile patient autonomy and informed consent - and therefore getting your patient to knowingly consent to being put on a placebo - with prescribing a "medicine" that you know has no pharmacologic effect? Or do you not have the same ethical basis for your practice?

I will end my involvement in this discussion by reiterating that if you want to be taken seriously, all you need to do is support you ideas with objective evidence. If you won't - or can't - do that, then all the good intentions in the world are as so much dust.

SS wrote, "today at one of our free clinics my patient was a 7 y/o boy whose chief complaint was a disabling phobia of wind." I'd try hypnosis first, or desensitization therapy, not aconite. As a marvelous ER doc I used to work with said, "If it's "all in his head, start with his head".

Doug wrote, "The publicâs emergency health care is in your hands? Scary.
NDs want the public's non-emergency care to be in in the hands of people who deny the germ theory of disease, believe in undetectable forces although those forces tend to fail properly blinded testing, overestimate the protective value of good life style, underestimate viruses and bacteria, and believe that molecules that aren't there any more can still have an effect? That's far scarier.

Doug wrote, "So you are claiming every PCP saw this outbreak? Sources. No, thanks to vaccines and quarantines the Pima County health department kept it from spreading. The San Diego outbreak also hospitalized some children, including a 2-month old infant who had the bad luck to be in the same waiting room as the index case ... an unvaccinated American child whose parents took him to Switzerland, a hotbed of naturopathy, where he encountered the virus.

Standard supportive care isnât treatment, but nice try. Since when is it NOT treatment? It's so embedded in medicine that it's assumed to be present.

I wrote, â... antibiotics are prescribed if the complications include a secondary bacterial infection, which is very likely because measles virus suppresses the kid's immune response.â

Doug answered, "Antibiotics are not used to treat measles, but it sounds like you would be one of those docs that would be prescribing antibiotics and aspirin for measles." That's not what I said. I said aspirin was contraindicated (medical jargon for "don't do it!") and I said antibiotics IF the complications include a secondary bacterial infection. You treat the bacteria with the antibiotics, and use supportive care (up to and including ventilators) for whatever the measles virus throws at you.

Doug wrote, "Since toddlers are drug users and have sexual partners too. Just because it CAN be transmitted by sex and IV drugs doesn't preclude other modes of spread. You get intra-household infection from infected adult to toddler ... then a lateral jump to the other toddlers with the sharing of toddler-appropiate body fluids.
Please see: http://pediatrics.aappublications.org/cgi/content/abstract/108/5/1123

By Tsu Dho Nimh (not verified) on 18 Apr 2009 #permalink

PalMD: help, I'm trapped in the spam filter, really]!!! It's a bit like Tron actually, and the neon is killing my eyes [not really].

-r.c.

I want to go on record that I believe in the germ theory. I also want to go on record that some people can clear a germ better then others due to individual factors. This is why we treat immune compromised patients and the elderly and or feeble differently when they are exposed to a germ. This is not a controversial stance but the generally accepted theory of disease. I would also say that there are some viruses and Germs that are so deadly almost no one can clear them regardless of the state of their health Rabies, HIV (unless a rare genetic anomaly), eboli the bubonic plague to name a few.

Hepatis B 1996: 54 Cases of disease reported to cdc in 0-1 age group (3.9 million births) 0.001% incidence. CDC.

So much for your "toddler-appropiate body fluids" theory.

Vaers reports 1996---1080 adverse reactions 0-1 age group with 47 deaths.Vaers.

Lupus: Tudela & Bonal (1992); Mamoux & Dumont (1994); Guiserix (1996)

Arthritis, including polyarthritis and rheumatoid arthritis: Christan & Helin (1987); Hachulla et al (1990); Rogerson & Nye (1990); Biasi et al (1993),(1994); Vautier & Carty (1994); Hassan & Oldham (1994); Rheumatic Review (1994); Gross et al (1995); Pope et al (1995); Cathebras et al (1996); Soubrier et al (1997);

Guillain Barre Syndrome GBS: Shaw et al (1988), Tuohy (1989)

Demyelinating disorders such as optic neuritis, Bell's Palsy, demyelinating neuropathy, transverse myelitis and multiple sclerosis: Shaw et al (1988); WHO (1990); Reutens et al (1990); Herroelen et al (1991); Nadler (1993); Brezin et al (1993); Mahassin et al (1993); Kaplanski et al (1995); Baglivo et al (1996); Marsaudon & Barrault (1996); Berkman et al (1996); Waisbren (1997)

Diabetes mellitus: Poutasi (1996); Classen (1996)

Chronic fatigue: Salit (1993); Delage et al (1993)

Vascular disorders: Fried et al (1987); Goolsby (1989); Cockwell et al (1990); Poullin & Gabriel (1994); Mathieu et al (1996); Graniel et al (1997)

Hep B in infants still necessary Tsu Dho Nimh?

Hep B is an often asymptomatic infection. The longer it persists in the system, the higher the risk of liver failure and liver cancer, a major cause of death around the world given the ubiquity of the virus.

Giving Hep B vaccine later in life is putting people at higher risk for the development of liver cancer.

Complications from the HBV vaccine are vanishingly rare, despite your cherry picked studies.

Please study some epidemiology, virology, and pathophysiology before making idiotic comments.

If you are big enough please provide sources for your claims. You demand them yourself.

To catch up to the rest of the class, you may want to start here: http://content.nejm.org/cgi/content/full/346/22/1682
http://content.nejm.org/cgi/content/short/359/14/1486
http://content.nejm.org/cgi/content/short/350/11/1118
http://content.nejm.org/cgi/content/short/351/27/2832

I'm sorry that it may be behind a paywall, but that isn't really my problem.

  • prevalence about 350,000,000
  • liver failure, cirrhosis, or cancer in 15-40% of those infected
  • The earlier infection takes place, the more likely for it to become persistent.
  • This strategy [vaccination] has reduced the overall annual incidence of acute HBV infections in the United States (from 8.5 cases per 100,000 persons in 1990 to 2.8 per 100,000 in 2002, a decrease of 67 percent
  • "HBV infection accounts annually for 4000 to 5500 deaths in the United States and 1 million deaths worldwide from cirrhosis, liver failure, and hepatocellular carcinoma"
  • "Concerns about potential disease risks (including vaccine-induced autoimmune diseases, diabetes, chronic fatigue syndrome, demyelinating disorders, optic neuritis, and other syndromes7,31,32) have not been supported by scientific data. Multiple studies have found no evidence of the initiation, relapse, or exacerbation of demyelinating disorders in association with hepatitis B vaccination."
  • PalMD
    What kind of an immune response can a child of that age mount when vaccinated against Hep B? What age are we talking about for the first shot? I really want to know. My understanding of an infants ability to mount an immune response is limited and one of the reasons we encouage breast feeding. do you think a mother immunized against HEP B who breast feeds would supply the child with any immune protection from her own vaccinations? If that protection is minimal what kind of immune response ie antibody response could a new born create IgG only?, and can't that be supplied by mother's milk? Genuinely want to know your opinion as I am curious and want to learn more about this subject.

    Your understanding is clearly insufficient, dw. If you read the studies and the data, you can clearly see that the vaccination is protective. In fact, early childhood/infant vaccination has ALWAYS been a clearly effective intervention. This nonsense about "What kind of an immune response can a child of that age mount" is just that...nonsense. It was asked, quite appropriately, a couple of decades ago, and answered. The fact that you don't understand or know is simply a fallacious argumentum ad ignorantium

    Docwimz ... One shot, at birth, containing ONE protein from the Hepatitis B virus is enough to get a protective immune response.

    That's sufficient, and well covered in the literature

    By Tsu Dho Nimh (not verified) on 18 Apr 2009 #permalink

    @Docwimz: You're welcome to refer to
    PEDIATRICS, 109 (1), 124-129 DOI: 10.1542/peds.109.1.124

    Additionally I've been writing an entire series on vaccines over at my blog, it's aimed at parents rather than doctors, because this material should have been covered in your preclinical medical training, but you're welcome to read it here.

    Okay, so everyone is still mocking SS and now docwimz, yet I see no answers to SS's question. Admittedly, I am just honestly curious, in a non-confrontational way, as to how the MDs on here would deal with that particular patient. Docwimz, thus far, is the only one to offer input. And, what do you all do? Tear him/her down as to why he/she does not practice homeopathy, instead of noting how Docwimz offered a valid suggestion: refer the patient out to a therapist who deals with phobias.

    Furthermore, the point of whether or not it is ethical to give a "sugar pill" to the child as treatment was made. Well, my guess in this situation is that the parents have already seen MDs, have already been given the normal drugs, and since those didn't work the parents are now looking for an alternative solution. You can debate that all you want, but you know that that is likely the situation at hand. In our clinic, we are very clear to patients (and parents) as to what homeopathy is (or acupuncture, hydrotherapy, etc.). Do we tell the patients, "Here, take this sugar pill, hopefully you will get a placebo effect"? No, of course not. But, I am also willing to assume that after running the gauntlet of drugs on their kid, to no avail, if you DID say that to the parents, while also telling them that there are no side effects to the treatment, they would probably still be willing to give it a try. Why not? Nothing else has worked. Of course, I am basing everything I just said on guessing and assumptions, which is not scientific. But, I don't have the entire case in front of me, nor did I interact with the patient and parents myself. I'm just speaking from the experience of peds shifts I've been on and the desperate parents I've seen.

    But come on people, for my own future patients, I'd like to see the MDs answer.

    Oh, and you can all tear me down now for what I just said (fresh meat thrown to the pack of dogs, as I see it), and that's fine. Just know that I will be off studying for finals and though I may read it, I likely won't respond:)

    By Andrea LaCoss (not verified) on 18 Apr 2009 #permalink

    I'm neither a pediatrician nor a psychiatrist. It is very far outside my expertise.

    In addition to being able to practice the full scope of your specialty, you have to be willing to admit when something is beyond your scope of practice.

    Well, my guess in this situation is that the parents have already seen MDs, have already been given the normal drugs, and since those didn't work the parents are now looking for an alternative solution.

    I'm not a psychiatrist. However I happen to be studying for my psychiatry shelf exam right now, and have seen this specific situation (not with wind persay).

    The answer below is a 3rd year med student level response, if a psychiatrist would like to correct me, please do!.

    To be clear: In real life, I'd refer this person to a psychiatrist, I wouldn't handle it as a PCP or in the emergency room (since the discussion has been primarily about PCPs, and I will be going into emergency medicine)

    Firstly, "the normal drugs" is not specific enough to have meaning here. The reason we can't give a realistic answer is that we don't know enough about the patient, or the patients history. Among other things, we don't know what drugs were tried (SSRI is not specific enough, they really aren't all the same) or for what lengths of time they were tried.

    More importantly, we don't have a real diagnosis here, the history given isn't detailed enough to decide which DSM IV diagnosis is appropriate - even though it comes billed as "phobias". If the diagnosis really was phobia, it would be odd for the psychiatrist to have prescribed SSRIs and benzos initially - that would be more likely if the outside psychiatrist thought the diagnosis was panic disorder. This is my differential of what it could be:
    1) Panic disorder with agoraphobia.
    2) Specific phobia, environmental -of wind as billed.
    3) OCD: with recurrent intrusive thoughts relating to wind, and question compulsions based on further history, possibly including locking self in bathroom. - this diagnosis is least likely but the history given is inadequate to rule this out.

    For Panic Disorder with agoraphobia

    General good guidelines that I've seen -This is explicitly not evidence based medicine, I have no citation and am going entirely on what i've seen practiced this month - is starting with paroxetine or sertraline, very low doses to start.
    Anecdotally, the most common mistake I've seen is that someone will start an SSRI at too high a dose, leading to some activating side effects that make the patients feel worse. So if the outside psychiatrist had started an SSRI too high, i'd start one at a much lower dose. Probably 5mg, and going up very slowly.

    Adjunctively one can refer for cognitive behavioral therapy, insight oriented psychotherapy, or family therapy. Note that these are all distinct and different from desensitization as described by docwimz

    For Specific phobia

    Actually docwims isn't unreasonable here, if the diagnosis is appropriately specific phobia, than desensitization therapy is a good idea, especialyl if one adjunctively prescribes beta blockers. SSRIs generally aren't as effective, but paroxetine is approved for this condition as well, start low, go slow here as above.

    For OCD

    Least likely diagnosis, but we can't rule this out without specific history from the child of what's going on in his head when he has these episodes. Panic attacks are less likely with this one.

    SSRIs are treatment of choice, the textbook answer is that one needs to go to very high doses to have an effect.
    Behavioral therapy - exposure and response prevention, is the other textbook answer.

    As a last resort, if the above fails, in OCD one can also use electroconvulsive therapy.

    Please note, none of the above is meant to be taken as medical advice. Those are the answers to SS's question, if it had been asked to me as a boards question. I will not diagnose or treat a patient who I haven't seen, and you shouldn't accept treatment taken from someone who hasn't seen you.
    I really wasn't planning on answering but every naturopath in the building was whining about it.

    PLease note the difference between my answer and the others here. I didn't jump straight to treatment, I didn't assume things about the outside psychiatrist, I started from the patient and story in front of me. I built a differential diagnosis, and made treatment plans based on it.

    Docwimz answer may not have been incorrect for specific phobia, and kudos for saying you wouldn't use aconite, but my problem with your answer is that you didn't dig into the story to find a real diagnosis!

    Thank you, Whitecoat Tales. I was the only one "whining" about it, and I was only "whining" about it b/c there was too much mockery occurring and not enough answers.

    I completely agree that we do not have a detailed enough history to diagnose or offer true suggestions. Even on a homeopathic level (which I know you don't agree with), there was not NEARLY enough information given to ascertain that Aconite was the correct choice.

    And PalMD, I COMPLETELY agree with your comment on knowing when something it outside your scope of practice. Knowing the limitations of your medicine (ND, MD, DO, whatever) is something that I feel is an extremely important responsibility of all physicians.

    Back to studying...

    By Andrea LaCoss (not verified) on 18 Apr 2009 #permalink

    I'm working on my kayak. No time for a long post. Some thoughts in no particular order:

    When a patient isn't getting better and you're the third or fourth doc the family has seen, it's time to go back to the drawing board. Doubt everything, but don't convey an attitude of suspicion.

    It might not be the wind. It might not be a phobia. The child may not be the one who needs help.

    Get all relevant prior records.

    Characterize the chief complaint thoroughly - duration, intensity, frequency, mitigating factors, etc. Review the history, noting periods of time when things were at their best and when things were at their worst. Any environmental correlates with those extremes?

    Everyone has a second or third degree relative with a few screws loose. So the grandmother and cousin's phobias don't add to our diagnostic confidence.

    Ask the family, daycare, babysitter, teacher to keep a log for an appropriate period of time, depending upon the frequency of attacks. This will help corroborate the history and will serve as a treatment baseline.

    Consider medical conditions that might present with paroxysms of distress --e.g., pheochromocytoma, supraventricular tachycardia, reactive airways, caffeine or stimulant use, thyroiditis, abdominal migraine, seizure disorder, etc.

    Latency age kids often develop phobias or compulsions that last several months to a couple of years. Developmentalists invent just-so stories to explain this. I haven't the time. But the beautiful thing here is this: there's a good chance the kid will make great progress no matter what you do.

    Functional behavioral analysis. Antecedents and consequences. A seven year old who "locks" himself in the bathroom for hours on end has likely figured out how to have pseudo-panic attacks.

    When a seven year-old runs the household, consider that mommy and daddy may not be working as a team.

    Assuming the child has an anxiety disorder and nothing else and the parents are not insane, I'd maximize non-biological interventions. Start by setting realistic goals. If the kid is having attacks every day and isn't going to school, a first goal might be to get the kid onto school property for X minutes.

    Find a cognitive-behavioral therapist who works with anxiety symptoms in kids. Consider a course of systematic desensitization combined with a structured behavioral program involving both the school and the home.

    If the parent balks at therapy and demands a magic potion or vitamin, you're in trouble.

    #197 I did not know this was suppose to be answered as a board question Whitecoats. I thought SS wanted honest advice??
    No I am not convinced that homeopathy has any validity to it. And I wish it was not required in ND school except as a voluntary elective. I would have rather spent my time getting more in dept courses in the sciences, and there were other people in my graduating class who groaned when they were forced to take it. I had problems with certain ND courses I was forced to take in ND school, but I was even more bothered by the MD medicine I saw that had the view disease=drug ABC. (this was 6 years ago by the way). However, as the years go by I find myself conflicted. If I had it to do over again I might have chosen to be an MD with CAM training. Sorry if this offend anyone out there (probably everybody out there)but its the truth. The reason being because there science training and Emergency medicine is better and I see the gaps in my own education. They are stricter, but I think, given what we do that is valid.
    Regarding my validation that homeopathy might help, I did not give the child the homeopathy, the ND did in the case. Personally, I would have referred the child to a therapist specializing in phobias and skipped the homeopathy all together. I would have talked to the mom, and the child, and taken notes, and I would have, with the mother's permission, sent them off to the person who was handling the case. I was not justifying placebo use in my own practice. I would not be too quick with the SSRI's the child had a bad response already, and it might not be needed if he can get immediate help with a good therapist. However, if not I agree with whitecoats on this.
    PALMD
    My understanding is that the vaccination is somewhat protective but that there is a limit in the immune response in a newborn; and for this reason it has benefits in newborns. I am NOT anti vaccines by the way.
    As regards immune response, in children I thought this was currently being debated by WHO when the best time for the series of vaccines should start and stop. I was trying to get info on infant immune development and how the current series is being updated. By the way, as a third year ND student I volunteered giving vaccines in central America to first years via the Red Cross in Nicaragua and loved it. I also saw how water borne illness and lack of immunization is killing children in the third world.
    My question regarded development of the immune response in children. I wasn't trying to argue just get more data. You never answered my question about breast feeding and passive immunity. Please refer to the information on the WHO web site regarding breast-feeding and immune support in non-immunized populations. Please clarify for me palMD your saying the immune response in a newborn is sufficient or enough or the same? Your saying that IgG is enogh right? Please could you send me the data and I will pass it on to other ND's THANKS. Because I will readily admit you probably know more then me about immunology and I am genuinely trying to learn something here. Thanks for the teachings and the debate :-)

    Docwimz would you really give medical advice over email or on a web forum. I don't believe it's at all appropriate to give medical advice to a patient you don't have an ongoing relationship with without seeing them, and doing a legitimate history and physical yourself.

    Given the amount of information SS was able to give us, a "board question" style answer is as much as was feasible and ethical to give.

    My question regarded development of the immune response in children. I wasn't trying to argue just get more data

    Docwinz I specifically gave you a journal reference addressing this topic, among other common concerns that parents may bring up.

    A 50 year old woman shows up with a red line running up the vein in her arm and says it was from a cat bite.

    What does a naturopathic shaman suggest?

    Vitamins, distilled water, chakra alignment, aura fiddling...?

    "I want to go on record that I believe in the germ theory."

    It is a sad state of affairs when this has to be said.